DNB Obstetrics
DNB Obstetrics
DNB Obstetrics
c) Hysteroscopy
d) LH surge
Correct Answer - C
Hysteroscopy REF: 12th edition of Novak's Gynecology p. 408
METHODS TO DOCUMENT OVULATION:
Luteinizing Hormone Monitoring Documentation of the LH surge
represents a remarkably reproducible method of predicting
ovulation. Ovulation occurs 34 to 36 hours after the onset of the LH
surge and about 10 to 12 hours after the LH peak
Basal Body Temperature The least expensive method of confirming
ovulation is for the
patient to record her temperature each morning on a basal body
temperature (BBT) chart.
Midluteal Serum Progesterone
Ultrasound Monitoring Ovulation can also be documented by
monitoring the development of a dominant follicle by ultrasound until
ovulation takes place
A ferning pattern is seen when periovulatory cervical mucus is
spread and dried on a microscope slide.
2. Minimum hCG level that a urine pregnancy
test can detect is?
a) 5 m IU/ ml
b) 10-20 m IU/ ml
c) 20-30 rn IU/ ml
d) 35 m IU/ ml
Correct Answer - A
5 m IU/ ml REF: Danforth's Obstetrics and Gynecology, 10th Edition
page 4, Current OB/GYN > Chapter 9. Normal Pregnancy & Prenatal
Care > Normal Pregnancy >
URINE PREGNANCY TEST:
Sensitive, early pregnancy test measure changes in levels of hCG.
There is less cross-reaction with luteinizing hormone (LH), follicle
stimulating hormone (FSH), and thyrotropin, which all share
common a subunit with hCG, when the /3 subunit of hCG is
measured. hCG is produced by the syncytiotrophoblast 8 days after
fertilization and may be detected in the maternal serum after
implantation occurs, 8-11 days after conception. hCG levels peak at
approximately 10-12 weeks of gestation. Levels gradually decrease
in the second and third trimesters and increase slightly after 34
weeks. The half-life of hCG is 1.5 days. After termination of
pregnancy levels drop exponentially. Normally, serum and urine
hCG levels return to nonpregnant values (< 5 m U/mL) 21-24 days
after delivery.
hCG is measured in milli-international units per milliliter (m IU/ml)
The detection of greater than 35 m IU of human chorionic
gonadotropin (hCG) in the first morning void has a very high
specificity for pregnancy
/3 Hcg in m IU/m1 Result
Under 5 m IU/ml Negative- Not pregnant
Between 5-25 m "Equivocal"- Maybe pregnant may not
IU/ml: be- Repeat test
Over 25 m IU/ml Positive- Pregnant
3. The shortest diameter of fetal head is :
a) Biparietal diameter
d) Bitemoral diameter
Correct Answer - D
Ans. is d i.e. Bitemporal diameter
Remember friends : Transverse diameters of the feta! skull are
smaller than Aritero posterior diameters Amongst the given options :
Biparietal and Bitemporal diameters are transverse diameters,
whereas suboccipito frontal and occipito frontal are anteroposterior
diameters.
Now, the choice is between bitemporal and biparietal diameters.
For memorizing this : learn a mnemonic 'Miss Tina so pretty', where
transverse diameter are arranged in ascending order of their size.
Miss Bimastoid diameter = 7.5 cms.
Tina Bi temporal diameter = 8 cms.
So Super subparietal diameter = 8.5 cms.
Pretty Bi parietal diameter = 9.5 cms. So,
our answer is bitemporal diameter (8 cms.)
Remember
Extra Edge :
Anteroposterior diameters of fetal skull
The longest diameter of fetal skull is mento vertical diameter (14
cms).
Longest diameter of pelvis — Transverse diameter
of inlet and antero posterior diameter of
anatomic outlet.°
Shortest diameter of pelvis — Posterior sagital
diameter of outlet°
Longest diameter of inlet — Transverse diameter°
Shortest diameter of inlet — Sacrocotyloid°
Longest AP diameter of inlet — Diagonal conjugate°
Shortest AP diameter of inlet — Obstetric conjugate°
Only AP diameter measured clinically Diagonal conjugate°
Attitude of the head
Diameters
Presentation
Suboccipito-bregmatic - 9.5cm (3 3/4")
extends from the nape of the neck to the
centre of the bregma
Complete flexion
Suboccipito-frontal - 10cm (4") extends
Vertex
from the nape of the neck to the anterior
Incomplete
end of the anterior fontanelle or centre of
flexion
the sinciput
Vertex
Occipito-frontal - 11.5cm (41/2") extends
Marked
from the occipital eminence to the root of
deflexion
the nose (Glabella).
Vertex
Mento-vertical - 14cm (51/2") extends
Partial extension
from the mid point of the chin to the
Brow
highest point on the sagittal suture.
Incomplete
Submento-vertical - 11.5cm (41/2")
extension
extends from junction of floor of the
Face
mouth and neck to the highest point on
Complete
the sagittal suture.
extension
Submento-bregmatic - 9.5 cm (33/4")
Face
extends from junction of floor of the
mouth and neck to the centre of the
bregma.
4. By which day after fertilization, is placental
circulation established :
a) 11th day
b) 13th day
c) 15th day
d) 17th day
Correct Answer - D
17th day
Development of the placental villi:
While the embryo is nourished in the first weeks through simple
diffusion, later, due to its rapid growth, it needs a more powerful gas
and nutrient exchange system.
This is made possible by the development of the utero-placental
circulation system in which the circulation systems of the mother and
of the embryo get closer together, thus allowing an exchange of
gases and metabolites via diffusion.
Through the lytic activity of the syncytiotrophoblast the maternal
capillaries are eroded and anastomose with the trophoblast lacunae,
forming the sinusoids.
At the end of the pregnancy the lacunae communicate with each
other and form a single, connected system that is delimited by the
syncytiotrophoblast and is termed the intervillous space.
1.Cytotrophoblast
2.Syncytiotrophoblast
3.Spaces between syncytiotrophoblast (Lacunae)
4.Maternal vessel
Maternal vessel, eroded by the ST,which form the maternal
sinusoids through communication with the lacunae.
Between the 11th and 13th day cytotrophoblast cells penetrate into
the cords of the syncytiotrophoblast creating the primary trophoblast
villi
1.Cytotrophoblast
2.Syncytiotrophoblast
After the 16th day the extra-embryonic mesoblast also grows into
this primary trophoblast villus, which is now called a secondary
villus and expands into the lacunae that are filled with maternal
blood. As was already mentioned, the ST forms the outermost layer
of every villus.
1.Extra-embryonic mesoblast
2 Cytotrophoblast
3 Syncytiotrophoblast
At the end of the 3rd week the villus mesoblast differentiates into
connective tissue and blood vessels. They connect up with the
embryonic blood vessels. Villi that contain differentiated blood
vessels are called tertiary villi
1 Extra-embryonic mesoblast
2 Cytotrophoblast
3 Syncytiotrophoblast
4 Fetal capillaries
5. The shortest diameter of fetal skull is :
a) Sub occipito frontal
b) Sub mentobregmatic
c) Mento vertical
d) Submentovertical
Correct Answer - B
Sub mentobregmatic
6. Hormone responsible for decidual reaction
and Arias stella reaction in ectopic
pregnancy is :
a) Oestrogen
b) Progesterone
c) HCG
d) HPL
Correct Answer - B
Ans. is b i.e. Progesterone
A single progesterone measurement can be used to establish that
there is a normally developing pregnancy with high reliability.
A value exceeding 25 ng/mL excludes ectopic pregnancy with 92.5%
sensitivity.
Arias stella reaction
Arias stella reaction is characterized by adenomatous change of the
endometrial glands.
Cells loose their polarity, have hyperchromatic nucleus, vacuolated
cytoplasm and occasional mitosis.
The reaction is seen in ectopic pregnancy (in 10-15% cases) and
indicates blightening of conceptus be it intrauterine or extrauterine.
(therfore it is not specific for ectopic pregnancy)
It occurs under the influence of progesterone.°
Decidual reaction
Under the hormonal effect of ectopic pregnancy (i.e. oestrogen,
progesterone and chorionic gonadotrophin) the endometrium
hypertrophies and is converted to decidua similar to normal uterine
pregnancy.
The decidua has all the characteristics of intrauterine
pregnancy except it does not have chorionic villi.°
The decidual reaction occurs under the influence of estrogen,
progesterone and HCG.°
So, hormone which is common to both Arias stella reaction and
decidual reaction is progesterone which is our answer of choice.
7. During pregnancy the maternal blood
volume increases by nearly :
a) 5 – 10%
b) 15 — 20%
c) 50%
d) 70%
Correct Answer - C
Ans. is c i.e. 50%
Friends, we have dealt with hemodynamic changes during
pregnancy in detail earlier but here lets quickly revise a few changes
asked quite often.
Most important physiological changes during pregnancy (for 11th
hour revision)
Net weight gain in pregnancy is 11 kg (24 lb).°
Blood volume T's by 40-50%.°
Blood volume reaches maximum at 30-32 weeks.°
Plasma volume T's by 50%.°
RBC volume T's by 20-30%.°
TLC is increased, specifically Neutrophilic leucocytosis occurs.°
Total plasma proteins increase.°
Plasma protein concentration decreases.°
Albumin level decreases, globulin levels (specially IgA) slightly
increase.°
Fibrinogen levels increase by 50%.°
ESR is increased.°
All clotting factors except XI and XIII increase.°
Platelet count decreases (Gestational Thrombocytopenia).°
Cardiac output (stroke volume) increases to 40-50% by 30-34
weeks.°
Peripheral vascular resistance decrease.°
Diastolic BP decreases.°
Uterine blood flow increases near term.°
Plasma insulin levels increase.°
S. Prolactin levels increase.°
Vital capacity and respiratory rate remain constant.°
Renal plasma flow and GFR increase.°
All LFT'S remain unchanged, with exception of alkaline phosphatase
which increases (specially heat stable alkaline phosphatase
produced by placenta).°
8. Minimum number of antenatal visits :
a) 3
b) 1
c) 5
d) 6
Correct Answer - A
3
9. Quickening can be felt at ...... weeks :
a) 14 weeks
b) 15 weeks
c) 16 weeks
d) 19 weeks
Correct Answer - C
16 weeks
10. What is not included in active
management of third stage of labour:
a) Early cord clamping
b) Uterine massage
c) Use of oxytocin
Correct Answer - A
Ans: A. Early cord clamping
Duration
Stage Starts/ Primi- Multip-Abnormalities
Ends parousarous
Prolonged third
3rd Delivery 0-0.5 0-0.5
stage:
hour hour • Failure to deliver
of infant
placenta within 30
to
minutes
delivery
Cause: Consider
of
placenta
placenta.
percreta/accreta/increta
Management:
• IV oxytocin
• If oxytocin fails,
attempt manual
removal
• Hysterectomy may
be
needed
11. Labour is termed as precipitate if it occurs
under:
a) 1 hour
b) 2 hours
c) 1/2 hours
d) 4 hours
Correct Answer - B
2 hours
12. The most common site of puerperal
infection is :
a) Episiotomy wound
b) Placental site
c) Vaginal laceration
d) Cervical laceration
Correct Answer - B
Placental site
13. Cervical incompetence is characterised by
:
a) Cerclage operation done
d) All
Correct Answer - D
Ans. is a, b and c i.e. 2nd trimester abortion; Premature rupture of
membrane; and Cerclage operation done
Causes of cervical incompetence
Congenital: Uterine anomalies
Acquired: Following D and C operation;Induced abortion by D and E
operation; Vaginal operative delivery through an undilated cervix;
amputation of the cervix or cone biopsy.
Other: Multiple gestations or prior preterm birth.
14. In cervical incompetence, encerclage
operation done are
a) Mc Donald operation
d) a and b
Correct Answer - D
Ans. is a and b i.e.Mc Donald operation and shirodkar operation
Friends, Cervical incompetence is an important topic. Many
questions have been asked on it time and again. So I am giving all
its details.
Cervical incompetence
Cervical incompetence is -l-1,acterised by r2inins( cervical dilatation
in the second° or early third trimester° with ballooning of the amniotic
sac into the vagina°, followed by rupture of membranes and
expulsion of a usually live fetus. The usual timing is 16 to 24 weeks.
Aetiology :
Congenital ?
- Developmental weakness of cervix.
- Associated with uterine anomalies like septate uterus.
- Following in utero exposure to diethyl stilbestrol.
Acquired due to previous cervical trauma ?
Forcible dilatation during MTP and D & C.
- Conisation of cervix.
- Cauterisation of cervix.
Amputation of cervix or Fothergill's operation.
Diagnosis :
History: The typical history of painless rupture of membranes°
followed by the quick delivery of a live fetus in midtrimester is very
suggestive.°
Non pregnant state: The internal os allows the passage of a No. 8
Hegar's cervical dilator or Foley's catheter filled with 1 ml water
without resistance.°
Premenstrual Hysterocervicography will show the typical funneling of
the internal os°.
In Pregnancy:
- Transvaginai ultrasound is the ideal method to follow up and detect
early incompetence.
- The normal cervical length at 14 weeks is 35 - 40 mm and the
internal os diameter is less than 20 mm. A cervical length less than
30 mm and an internal os diameter more than 20 mm is suggestive
of cervical incompetence.
- Funneling of the os.°
Note: Funneling is the ultrasound finding of herniation of the fetal
membranes into the upper part of the endocervical canal.
Management : The treatment is surgical by a cervical cerclage.
Time of operation : Cervical cerclage is usually delayed up to 12 - 14
weeks so that miscarriage due to other causes can be eliminated or
it should be done atleast 2 weeks earlier than the lowest period of
earlier wastage (but never earlier than 10 weeks).
Sonography should be done prior to cerclage to confirm a live fetus
and to rule out anomalies.
Procedures
Mc Donald's operation Shirodkar's operation
— It has good success rate & less blood loss.
— The most commonly performed procedure now-a-days.
— The Shirodkar operation is technically more involved & takes
longer to perform.
Abdominal cerclage : Indications of transabdominal cerclage:
Women with incompetent cervix due to severe trauma to cervix such
as deep laceration. extensive conization or repeated LEP for
treatment of Ca in situ.
H/O repetitive 2nd trimester loss and failed vaginal cerclages.
It is also indicated in women with 2nd trimester losses and anatomic
It is also indicated in women with 2nd trimester losses and anatomic
impossibility to place a vaginal cerclage.
Removal of Stitch : The Stitch should be removed at 37 — 38 week
or earlier if labour pain starts or features of abortion appear.
Contraindications
— Intra uterine infection
- Ruptured membranes
- H/o vaginal bleeding
- Severe uterine irritability
— Cervical dilatation > 4 cms
Complications
— Chorloamnionitis Rupture of membranes
— Preterm labour
— Necrosis of cervix
Rupture uterus
15. Blighted ovum is :
a) Synaptic knobs
b) Avascular villi
c) Intervillous hemorrhage
d) None
Correct Answer - B
Ans. is b i.e. Avascular villi
According to the clinical and echographic findings, it is
possible to separate early pregnancy losses into two groups :
alighted ova : Those early pregnancy losses in which foetal
development is not observed with ultrasound (so that only a
gestational sac is present with or without a yolk sac) and fetal tissue
is absent on histologic examination of the products of conception.
• Early Fetal demise Those early pregnancy losses in which fetal
development is clearly observed by ultrasound and feta! tissue is
found on the histologic examination.
The difference between these two types of abortion is of
fundamental importance. The lack of development of fetal structures
defines a subset of abortions of genetic origin.
In contrast, the early interruption of fetal life is a complex
phenomenon with multiple etiologies. Therefore, the patients
with blighted ova do not require extensive work up, whereas
patients who have aborted cytogenetically normal fetuses need
an extensive search for non genetic factors responsible for the
pregnancy loss.
16. The commonest chromosomal
abnormality in early spontaneous
abortions is :
a) Monosomy
b) Autosomal trisomy
c) Triploidy
d) Tetraploidy
Correct Answer - B
Autosomal trisomy
17. McDonald stitch is applied in the following
conditions except :
a) Incompetent os
b) Septate uterus
c) Placenta previa
Correct Answer - C
Placenta previa
18. Suction evacuation can be done up to:
a) 6 weeks
b) 10 weeks
c) 16 weeks
d) 18 weeks
Correct Answer - B
Ans. is b i.e. 10 weeks
19. Following is used in Ist trimester MTP :
a) Mifepristone
b) Laminaria tent
d) b and c
Correct Answer - D
b and c both
20. Recurrent abortion seen in all except:
a) Syphilis
b) TORCH
c) Rh incompatibility
d) Chromosomal abnormality
Correct Answer - B
TORCH
Recurrent miscarriage, habitual abortion, or recurrent pregnancy
loss (RPL) is three or more consecutive pregnancy losses at less
than (or equal to) 20 weeks of gestation or with a fetal weight of
<500 grams.
It affects approximately 1% to 2% of women
21. What is the treatment of choice of
unrupturece tubal pergnancy with serum
b-hCG titre 20001 U1 ml:
a) Single dose of methotrexate
c) Expectant management
d) Laparoscopic salpingostomy
Correct Answer - A
Ans. is a i.e Single dose of methotrexate
The lady in the question is presenting with unruptured ectopic
pregnancy with BhCG levels - 2000IU/L. So there is not doubt we
can manage the patient medically (i.e. option d - ruled out)
Expectant management as explained earlier is not better than
medical management as it carries the risk of rupture of ectopic
pregnancy (i.e. option C ruled out) and also it is done when riFICG
levels are < 1000 IUL
Now - The question arises whether we should give single dose MTx
or multidose methotrexate treatment. In the trials which have been
conducted - No difference was found in treatment duration, phCG
levels and side effects in single dose vs multidose methotrexate
therapy.
Single dose therapy is more commonly used becasue of simplicity. It
is less expensive, requires less invasive post therapy monitoring and
does not require leucovorin supplementation.
One trial has shown that multidose therapy has greater chance of
success than single dose but this is not supported by any other trial
so, single dose methotrexate is being used commonly. (So, it is our
option of choice.)
option of choice.)
Also know :
Single dose Medical treatment protocol for ectopic pregnancy.
Day 0 / Day I = Measure serum r3hCG, TLC, DLC, Liver
Function Test and Renal Function Test
Day 2 = Single dose Methotrexate 50 mg/m2 IM given
Day 4 = S. VilICG and counts repeated
Day 7 = S. 13hCG and counts repeated
Now if the decline in serum jhCG on Day 7 from day 4 is?15%/Fetal
cardiac activity present
Repeat methotrexate dose and begin new Day 1
Surgical treatment is indicated if rihCG levels not decreasing or fetal
cardiac activity persists
after three doses of methotrexate
22. True about complete hydatidiform mole is
:
a) Chromosome pattern is XX
d) All
Correct Answer - D
Ans. is a, b and c i.e. Chromosome pattern is XX; Enlarged
ovarian cyst occurs; and Associated with preeclampsia
The incidence of H. mole is maximum in oriental and south east
countries (maximum incidence is in
Philippines: 1 in 80 pregnancies) Le., it is more common in
developing countries option 'a ruled out).
H, mole can be categorized as either complete or partial mole
on the basis of Gross morphology, histopathology and
karyotype.
Complete H. mole - shows no evidence (If fetal tissue at all,
Complete hydatiform moles exhibit characteristic swelling and
trophoblastic hyperplasia. Most common karyotype is 46XX (10%
may have a 46XY karyotype).
The molar chromosomes are entirely of paternal origin, although
mitochondrial DNA is of maternal origin.
The complete moles arises from an ovum that has been fertilized by
a haploid sperm, which then duplicates
its own chromosomes called as Androgenesr. The ovum nucleus
may be either absent or inactivated.
Clinical features;
Symptoms:
Amenorrhea of varying duration followed by continuous or
intermittent brown or bloody discharge (usually not profuse) evident
by - 12 weeks.
Passage of vesicles per vaginum
Hyperemesis (due to the high levels of circulating HCG).
Signs:
Uterus:
- The fundal height of uterus is more than the period of
amenorrhea in 50% cases. In 35% it corresponds to the gestational
period and in the rest it may be smaller.
- Uterus is doughy in consistency (due to absence of amniotic
fluid).
- Fetal parts will not be felt.
- Fetal heart sounds will not be heard (even on doppler).
- External and internal ballottement can not be elicited.
Theca lutein cysts :
- May be felt in the ovaries in about 25 to 60%.
- Due to overstimulation of the luteal elements by the large
amounts of circulating HCG°
- Persistent trophoblastic disease is more likely in women with
theca lutein cysts.
Early onset preeclampsia
- When hypertension appears before 24 weeks, it is important to rule
out hydatidiform mole.
Thyrotoxicosis seen due to the thyrotrophin-like effect of HOG.°
Spontaneous expulsion occurs at around 18 weeks° and rarely
delayed beyond 28 weeks.°
Characteristics of partial H. mole
Some embryonic or fetal tissue identifiable.
Partial moles generally have a triploid karyotype (69 chromosomes);
the extra haploid set of chromosomes usually is derived from the
father
Chorionic villi of varying size with focal hydatidiform swelling,
cavitation, and trophoblastic hyperplasia whereas complete mole
shows diffuse hydatidiform swelling and trophoblastic hyperplasia.
Marked villous scalloping.
Prominent stromal trophoblastic inclusions.
- When a fetus is present in conjunction with a partial mole. it
generally exhibits the stigmata of triploidy, including growth
retardation and multiple congenital malformations such as
syndactyly and hydrocephaly.
23. Prognosis of Gestational Trophoblastic
Disease depends on all, except :
a) Number of living children
b) Blood group
c) Parity
Correct Answer - A
Ans. is a i.e. Number of living children
Gestational Trophoblastic Disease :
i) Low Risk (Good Prognosis) :
Disease is present < 4 months duration
Initial serum hCG level < 40,000 mIU/ml
Metastasis limited to lung and vagina
No prior chemotherapy
No preceding term delivery
ii) High Risk (Poor Prognosis) :
Long duration of disease (> 4 months)
Initial serum hCG level > 40,000 mIU/ml
Brain or liver metastasis
Failure of prior chemotherapy
Following term pregnancy
WHO score > 8
24. Most reliable diagnosis of hydatidiform
mole Is by:
a) HCG estimation
b) Amniography
c) Ultrasound
d) Absence of FHS
Correct Answer - C
Ultrasound
25. Which among the following is the smallest diameter of true pelvis?
a) Interspinous diameter
b) Diagonal conjugate
c) True conjugate
d) Intertuberous diameter
Correct Answer - A
Interspinous or bispinous diameter is the distance between the tip of the two ischial
spines. It measures 10.5 cm.
Diagonal conjugate is the distance between the lower border of symphysis pubis to the
midpoint on the sacral promontory. It measures 12 cm.
True conjugate or anatomical conjugate or conjugate vera is the distance between the
midpoint of the sacral promontory to the inner margin of the upper border of symphysis
pubis. It measures 11 cm.
Intertuberous diameter is the distance between the inner borders of ischial tuberosities. It
measures 11 cm.
Ref: Textbook of Obstetrics by D C Dutta, 6th edition, Page 91.
26. What is the total iron requirement during pregnancy ?
a) 500 mg
b) 750 mg
c) 1000 mg
d) 1500 mg
Correct Answer - C
Total iron requirement during pregnancy is 1000 mg, this requirement is mostly limited to
last 12 weeks.
Physiological changes in iron metabolism during pregnancy
In the second half of pregnancy the daily iron requirement is 6 - 7
gm.
Serum iron concentration decreases
Serum ferritin decreases
Serum total iron binding capacity increases
Percentage saturation (serum ferritin/ serum TIBC) decreases
Serum transferrin increases
27. Which of the following is NOT true about amniotic fluid index (AFI)?
a) It is calculated by adding the vertical depths of the largest
pocket in each of four equal uterine quadrants
Correct Answer - D
Amniotic fluid index (AFI) is calculated by adding the vertical depths of the largest pocket
in each of four equal uterine quadrants.
Peak AFI is seen at approximately 32 weeks followed by a steady decline until 42 weeks.
Ref: Cunningham F.G., Leveno K.J., Bloom S.L., Hauth J.C., Rouse D.J., Spong C.Y.
(2010). Chapter 21. Disorders of Amnionic Fluid Volume. In F.G. Cunningham, K.J. Leveno,
S.L. Bloom, J.C. Hauth, D.J. Rouse, C.Y. Spong (Eds), Williams Obstetrics, 23e.
28. Misoprostol has been found to be effective in all of the following except:
a) Missed abortion
b) Induction of labour
c) Menorrhagia
Correct Answer - C
Treatment of Menorrhagia involves administration of prostaglandin inhibitors. Misoprostol
(PGE1) is a prostaglandin and is therefore certainly not indicated for treatment of
metrorrhagia.
Ref: Textbook of Gynecology By D C Dutta, 6th Edition, Page 505
29. The following drugs/methods are used for cervical ripening, except?
a) Oxytocin
b) PGE
c) Stripping of membrane
d) Ergometrine
Correct Answer - D
Prostaglandins, oxytocin, and stripping of membranes are usually done for cervical
ripening. Other methods are Foley bulb catheter, early amniotomy, and late amniotomy.
Ergometrine is used in promoting of contraction and involution of uterus after delivery.
30. A hypertensive primigravida developed Sheehan’s syndrome after the delivery.
All of the following are seen with Sheehan's syndrome, EXCEPT:
a) Obesity
b) Amenorrhoea
c) Failure to lactate
Correct Answer - A
Severe intrapartum or early postpartum hemorrhage and associated hypotension is
followed by pituitary failure or Sheehan syndrome.
It refers to panhypopituitarism.
Abrupt, severe hypotension leads to pituitary ischemia and necrosis.
It is characterized by failure of lactation, amenorrhea, breast atrophy, loss of pubic and
axillary hair, hypothyroidism, and adrenal cortical insufficiency.
Also know:
The pituitary cell types are differentially sensitive to damage. For this reason, prolactin
secretion deficiency is the most common, followed by loss of gonadotropin and growth
hormone release, loss of ACTH production, and least commonly, by decreases in thyroid-
stimulating hormone (TSH) secretion.
Ref: Leveno K.J., Hauth J.C., Rouse D.J., Spong C.Y. (2010). Chapter 35. Obstetrical
Hemorrhage. In K.J. Leveno, J.C. Hauth, D.J. Rouse, C.Y. Spong (Eds), Williams
Obstetrics, 23e.
31. Undiagnosed or undetected ectopic pregnancy is a common cause of maternal
death during the first trimester. Most valuable diagnostic test in a case of
suspected ectopic pregnancy is:
b) Transvaginal USG
c) Progesterone measurement
d) Culdocentesis
Correct Answer - B
Transvaginal ultrasonography (TVUS) has the advantage of earlier and better
localization of the pregnancy, with less pelvic discomfort because the bladder is not
painfully distended.
The double-ring sign and the yolk sac must be identified to ensure that the pregnancy is
intrauterine.
When an intrauterine pregnancy is not visualized on TVUS and the hCG level exceeds
1000–2000 mIU/mL, suspicion for an ectopic pregnancy should be high.
Ref: Hill M.J., DeCherney A.H. (2013). Chapter 36. Imaging in Gynecology. In A.H.
DeCherney, L. Nathan, N. Laufer, A.S. Roman (Eds), CURRENT Diagnosis & Treatment:
Obstetrics & Gynecology, 11e.
32.
In which part of fallopian tube ectopic pregnancy will have longest survival?
a) Isthmus
b) Ampulla
c) Cornua
d) Interstitium
Correct Answer - D
If the implantation occurs in the antimesenteric border in the ampulla, the pregnancy may
continue a little longer time.
Earliest interruption occurs in the isthmial implantation and pregnancy may continue
upto 3-4 months in interstitial implantation.
Also know:
Isthmic rupture usually occurs at 6-8 weeks, the ampullary one at 8-12 weeks and the
interstitial one at about 4 months.
Ref: Textbook of Obstetrics by DC Dutta, 6th edition, Page 181.
33. Which of the following statements about partial mole is false?
a) Usually associated with Triploidy
Correct Answer - D
Partial mole cannot be diagnosed by ultrasonography at a very early gestational ages, before the
chorionic villi have attained vesicular pattern.
Ref: Williams Gynaecology, 1st Edition, Page 758; Novak's Textbook of Gynecology, 14th Edition,
Pages 1588, 1582; Obstetrics and Gynecology By Beckmann, 6th Edition, Page 360; Textbook of
Obstetrics By DC Dutta, 6th Edition, Page 201.
34. Which of the following is the most sensitive or gold standard test for assessing
HCG in maternal serum?
a) Radioimmune assay
b) ELISA
c) Latex test
d) Bioassay
Correct Answer - A
The radio-immune assay stands out to be the most sensitive test to measure the beta sub-
unit of human chorionic gonadotropin in which a quantitative assessment of levels more
than 5mIU/ml is read as positive.
Thus even very minute quantities of the hormone can be detected as following the
implantation of the blastocyst.
The antibodies used in this assay is highly specific for the beta subunit of the hCG.
Other than pregnancy, radioimmuno assay is also used for follow-up of cases like
hydatiform mole and choriocarcinoma.
Ref: Human Chorionic Gonadotropin (hCG) By Larry Cole, Laurence A. Cole, Stephen
Butle, Page 13-15; Textbook of Obstetrics, D C Dutta 6th edition, Page 61; Mudaliar and
Menon’s Clinical Obstetrics 9th edition, Page 53-55
35. What is the treatment of choice in pregnant women with polyhydramnios and
marked respiratory distress at 35 weeks of gestation?
a) Intravenous frusemide
b) Induction of labour
c) Amniocentesis
Correct Answer - C
In this clinical scenario amniocentesis is done to relieve the distress and to continue the
pregnancy upto term.
Slow decompression is done at a rate of about 500ml per hour.
Normally 1-1.5litres of amniotic fluid is removed.
Because of slow decompression there is less chance of accidental hemorrhage, but fluid
can reaccumulate soon.
So amniocentesis has to be repeated.
b) Both Assertion and Reason are true, and Reason is not the
correct explanation for Assertion
Correct Answer - A
a) Low platelets
b) Raised serum Na
Correct Answer - D
A serum uric acid level (biochemical marker of pre-eclampsia) of more than 4.5 mg/dl
indicates the presence of pre-eclampsia.
Blood urea level remains normal or slightly raised.
Serum creatinine level may be more than 1 mg/dl.
There may be thrombocytopenia and abnormal coagulation profile of varying degrees.
Hepatic enzyme levels will be increased.
Ref: Textbook of Obstetrics D C Dutta, 6th edition, Page 227.
38. 28 year old Nebisa has attended the obstetician for a routine anti-natal check up. It is
confirmed that she has twins. She is now referred for an USG.
b) Both Assertion and Reason are true, and Reason is not the
correct explanation for Assertion
Correct Answer - A
In dichorionic diamniotic twin gestation, the chorion and amnion of each twin reflect away from the
fused placenta to form the intertwine membrane.
Number of intertwine membrane here are 4.
This space in the intertwine membrane is filled by proliferating placental villi give rise to twin peak
sign.
Reference:
Textbook of Obstetrics By DC Dutta, 6th Edition, Page 206
39. Non Immune Hydrops Fetalis is caused by:
a) CMV
b) Parvovirus
c) HSV
d) HIV
Correct Answer - B
Non immune hydrops can be caused by a variety of causes. Among the infectious cause,
parvovirus can lead to non immune hydrops.
Hydrops fetalis refers to excessive accumulation of fluid in at least two fetal body cavities.
It can be of two types, immune and non immune.
Immune type result from hemolysis of fetal RBC by maternal IgG antibodies to fetal antigen.
Non immune type, is the result of any non antibody mediated process that lead to hydrops.
Ref: Fetal Medicine : Basic Science and Clinical Practice, 2nd Edition, Page 522; Ian
Donald's Practical Obstetric Problems, 6th Edition, Page 218; William's Obstetrics, 23rd
Edition, Page 627; Moffet's Pediatric Infectious Diseases : A Problem Oriented Approach,
4th Edition, Pages 377, 643.
40. A pregnant woman presents with red degeneration of fibroid, Management is:
a) Myomectomy
b) Conservative Rx
c) Hysterectomy
d) Termination of pregnancy
Correct Answer - B
In a pregnant female with red degeneration of fibroid, management is essentially
conservative treatment with analgesia and rest.
Ref: Textbook of Obstetrics By D.C.Dutta, 5th Edition, Page 1327 ;
Essentials of Obstetrics By Arulkumaran, 2004, Page 200
41. Treatment of choice for Intrahepatic Cholestasis in Pregnancy is:
a) Cholestyramine
c) Corticosteroids (Dexamethasone)
d) Antihistaminics
Correct Answer - B
Ref: Oxford Textbook of Medicine, 4th Edition, Pages 422 ; Textbook of Obstetrics By DC
Dutta, 6th Edition, Page 291 ; Bile Acids and Pregnancy By U. Leuschner, P. A. Berg, J.
Holtmeier, 2002, Page 36.
42. Which of the following is the best marker for intrahepatic cholestasis of
pregnancy?
b) Bile Acids
c) Bilirubin
d) Alkaline Phosphatase
Correct Answer - B
The rise in serum bile acids is sufficient in the third trimester is sufficient to confirm the
clinical diagnosis of idiopathic intrahepatic cholestasis of pregnancy which is also
accopanied by severe, intractable, generalized itching and sometimes clinical jaundice.
Ref: Schiff's Diseases of the Liver By Eugene R. Schiff, Willis C Maddrey, MD, Macp,
Willis C. Maddrey, Michael F. Sorrell, Pages 274-276; Medical Disorders in Obstetric
Practice By Michael De Swiet, 284 - 286.
43. Which of the following is the investigation of choice in cholestasis of
pregnancy?
Correct Answer - B
In patients with cholestasis of pregnancy the levels of serum bile acids are elevated 30-100
times the upper limit of normal. Levels of AST, ALT and alkaline phosphatase are only
mildly elevated.
Intrahepatic cholestasis of pregnancy:
The major risk factors for IHCP include multiparity, advanced maternal age, family history of
IHCP, and pruritus while on oral contraceptive medicines.
Patients often presents in the third trimester of pregnancy with pruritus beginning centrally
and spreading peripherally.
20% develop pruritus and jaundice. Ursodiol and cholestyramine are the first agents of
choice in the treatment of pruritus.
Ref: Ukomadu C. (2012). Chapter 8. Hepatic Complications of Pregnancy. In N.J.
Greenberger, R.S. Blumberg, R. Burakoff (Eds), CURRENT Diagnosis & Treatment:
Gastroenterology, Hepatology, & Endoscopy, 2e
44. A 23 year old female with type I diabetes presented with 6 weeks of
amenorrhea. Her urine pregnancy test turned to be positive. Her sugar levels
are not in control. The child may have all of the following abnormalities,
EXCEPT:
a) Chromosomal abnormalities
b) Caudal regression
c) Renal agenesis
d) Duplex ureter
Correct Answer - A
Diabetes is not associated with increased risk for fetal chromosomal
abnormalities.
Congenital malformations in infants of women with overt diabetes:
Caudal regression
Situs inversus
Spina bifida, hydrocephaly, or other central nervous system defects
Anencephaly
Cardiac anomalies
Anal/rectal atresia
Renal anomalies
1. Agenesis
2. Cystic kidney
3. Duplex ureter
Ref: Cunningham F.G., Leveno K.J., Bloom S.L., Hauth J.C., Rouse
D.J., Spong C.Y. (2010). Chapter 52. Diabetes. In F.G.
Cunningham, K.J. Leveno, S.L. Bloom, J.C. Hauth, D.J. Rouse, C.Y.
Spong (Eds), Williams Obstetrics, 23e.
45. Which of the following statements regarding fetal circulation is correct?
a) The liver and heart of the fetus receive blood with very high
oxygen saturation
d) The foramen ovale closes during the third trimester unless the
fetus has an atrial septal defect
Correct Answer - A
Since the liver is supplied by umbilical venous blood from the placenta, and the heart
and head receive blood before it has mixed with significant amounts of desaturated
blood, these important organs receive blood that is relatively high in saturated
oxyhemoglobin.
The high rate of blood flow at the placenta and the significant resistance of the placenta to
diffusion of oxygen result in blood in the umbilical vein that has a lower PO2 (30 mm Hg)
than the maternal mixed venous blood.
However, the left shift in fetal oxyhemoglobin concentration and the Bohr effect both act to
increase the transport of oxygen to fetal tissues.
The foramen ovale remains open until after birth and a significant portion of inferior
vena cava flow is shunted through it to the left.
1/3 of blood enters directly from right atrium to left atrium (bypassing right ventricle) through
foramen oval
The major portion of right ventricular output is shunted through the ductus arteriosus to the
aorta, not the lungs.
The net effect of these shunts in the presence of high fetal pulmonary vascular resistance is
very low fetal pulmonary blood flow.
At birth, these patterns normally are quickly changed to ex-utero patterns with high
pulmonary perfusion.
Ref: Barrett K.E., Barman S.M., Boitano S., Brooks H.L. (2012). Chapter 33. Circulation
through Special Regions. In K.E. Barrett, S.M. Barman, S. Boitano, H.L. Brooks (Eds),
Ganong's Review of Medical Physiology, 24e.
46. Which of the following Doppler findings in IUGR is associated with worst
prognosis?
a) Dicrotic notch
Correct Answer - B
Doppler screening is done in anatomically normal baby’s in which a growth retardation is
detected.
Normally the diastolic flow increases as pregnancy progresses.
Reduced or absent or reversed diastolic flow in the umbilical artery indicates fetal jeopardy
and poor perinatal outcome.
Doppler velocimetry findings in IUGR are elevated uterine artery systolic/diastolic ratio >
2.6 and or presence of diastolic notch.
Ref: Textbook of Obstetrics By D. C Dutta, 6th Edition, Page 463.
47. Misoprostol used in the induction of labour is an analogue of which of the
following type of prostaglandin?
a) PG E1
b) PG E2
c) PG I2
d) PG F2alpha
Correct Answer - A
Misoprostol is a methyl ester of PGE1.
Indications of Misoprostol:
It is used for cervical ripening.
Transvaginally it is used for induction of labour.
Uses of Prostaglandins in Obstetrics:
Induction of abortion
Termination of molar pregnancy
Induction of labour
Cervical ripening prior to induction of labour
Acceleration of labour
Management of atonic postpartum hemorrhage
Medical management of tubal ectopic pregnancy
Ref: Textbook of Obstetrics By DC Dutta 6th edn page 504.
48. During the delivery, it is necessary to cut an episiotomy. The tear extends
through the sphincter of the rectum, but the rectal mucosa is intact. How would
you classify this type of episiotomy?
a) First degree
b) Second degree
c) Third degree
d) Fourth degree
Correct Answer - C
A first-degree tear involves the vaginal mucosa or perineal skin, but not the underlying
tissue.
In a second-degree episiotomy, the underlying subcutaneous tissue is also involved, but
not the rectal sphincter or rectal mucosa.
In a third-degree tear, the rectal sphincter is affected.
A fourth-degree episiotomy involves a tear that extends into the rectal mucosa.
Ref:Cunningham F.G., Leveno K.J., Bloom S.L., Hauth J.C., Rouse D.J., Spong C.Y.
(2010). Chapter 23. Forceps Delivery and Vacuum Extraction. In F.G. Cunningham, K.J.
Leveno, S.L. Bloom, J.C. Hauth, D.J. Rouse, C.Y. Spong (Eds), Williams Obstetrics, 23e.
49. Which of the following is the chromosomal composition of the complete mole?
a) 46 XX
b) 45X0
c) 69 XXY
d) 69 XXX
Correct Answer - A
The chromosomal composition of complete moles is usually diploid and of paternal origin.
About 85 percent are 46,XX with both sets of chromosomes paternal in origin.
Termed androgenesis, the ovum is fertilized by a haploid sperm, which duplicates its own
chromosomes after meiosis.
The chromosomes of the ovum are either absent or inactivated.
In other complete moles, the chromosomal pattern may be 46,XY due to dispermic
fertilization.
Ref: Cunningham F.G., Leveno K.J., Bloom S.L., Hauth J.C., Rouse D.J., Spong C.Y.
(2010). Chapter.11. Gestational Trophoblastic Disease. In F.G. Cunningham, K.J. Leveno,
S.L. Bloom, J.C. Hauth, D.J. Rouse, C.Y. Spong (Eds), Williams Obstetrics, 23e.
50. In Downs syndrome, the 2nd trimester quadruple test done at 14-20 weeks
measure levels of all of the following, EXCEPT:
a) Alpha fetoprotein
b) HCG
c) Inhibin A
d) Progesterone
Correct Answer - D
The triple test (14 -20 weeks) measures the following three levels in the maternal serum:
Alpha-fetoprotein (AFP)
Human chorionic gonadotropin (hCG)
Unconjugated estriol (UE3)
a) 7-9 weeks
b) 10 weeks to term
c) 9-11 weeks
d) 13-15 weeks
Correct Answer - B
Chorionic villus sampling (CVS) is the second most common procedure for genetic
prenatal diagnosis. This procedure is routinely performed transcervically at about 10–12
weeks of gestation and transabdominally from 10 weeks to term. Limb reduction
defects are high when CVS was performed at less than 10 weeks of gestation.
c) Alcoholics
d) PIH
Correct Answer - C
Alcoholics
53. Placenta previa mouth is associated with
all of the following except :
a) Large placenta
b) Previous C. S. scar
c) Primigravida
Correct Answer - C
Ans. is c i.e Primigravida
Placenta previa is implantation of the placenta partially or completely
over the lower uterine segment. Damage to the endometrium or
myometrium due to previous surgery or infection can predispose to
low implantation and placenta previa.
Risk factors for placenta previa :
Prior surgery° (cesarean section / Myomectomy Hysterotomy)
Previous uterine curettage°
Endometritis°
Increasing maternal age (>35 years)°
Increasing parity°
Placental sire –increased (as in multiple pregnancy)
Placental abnormality –Succenturiate lobe
Smoking (due to defective decidual vascularisation)
Note :
The probability of placenta accreta and need for cesarean
hysterectomy is increased in patients with prior cesarean section
and placenta previa.
Smoking increases the risk of placenta previa by two fold times.
Previous cesarean section increases the risk of placenta previa by 4
fold time.
According to the latest edition of Williams—Women with otherwise
unexplained elevated screening level of maternal serum alpha feto
protein are at a greater risk of placenta previa.
54. True regarding PPH
a) Type B lynch suture used
Correct Answer - D
Ans. is a, b and c i.e. Type B lynch suture used; With new advances
both atonic and traumatic PPH can be reduced; More common in
multipara; and Associated with polyhydramnios
PPH is more common in multipara due to laxed abdomen and
associated factors like adherent placenta and anemia .
Overdistension of uterus as in multiple pregnancy, hydraminos and
large baby also lead to PPH .
Incidence of atonic and traumatic PPH can be reduced with new
advances or rather by intelligent anticipation, skilled supervision,
prompt detection and effective institution of therapy.
B lynch suture are used for management of PPH .
Mifepristone is not used in the management of PPH .
55. Most common cause of secondary PPH is
:
a) Uterine inertia
b) Retained placenta
c) Episiotomy
d) Cervical tear
Correct Answer - B
Retained placenta
56. Common cause of death in inversion of
uterus :
a) Neurogenic shock
b) Hemorrhage
c) Pulmonary embolism
d) Infection
Correct Answer - B
Hemorrhage
57. Correct statement about establishing the
chorionicity in twin pregnancy is:
a) Same sex rule out dichorionicity
Correct Answer - B
Ans. is b i.e. Twin peak in dichorionicity
Chorionicity is the most important factor that affects the
outcome in Twin gestation.
Diagnosed by ultrasound at 6 to 9 weeks of gestation. In
dichoronic twins there is a thick septum between the chorionic sacs.
It is best identified at the base of the membrane, where a triangular
projection is seen. This is known as lambda or twin peak sign.
Presence of lambda or twin peak sign indicates dichorionic placenta"
So it is clear that lambda / Twin peak sign clearly indicates
dichorionic placenta & is hence the correct option 'b'.
As far as other options are concerned.
Option a - Same sex rules out dichorionicity, this is incorrect
because
Twins of opposite sex are almost always dizygotic dichorionic but
same sex does not rule out dichorionicity. Option c - Thick
membrane is present in monochrionic twins
This is also incorrect because monochorionic means there is a
single chorion whereas dichorionic means there are 2 chorions so
obviously dichorionic membrane will be thick.
"Monochorionic pregnancies have a dividing membrane that is
so thin, it may not be seen until the second trimester. The
membrane is generally less than 2mm thick & magnification
reveals only 2 layers (of
amnion)"
Option d – Chorionicity is best detected after 16 weeks –
Again this statement is incorrect because the best time to detect
chrorionicity by USG is between 6 to 9 weeks.
58. At 28 weeks gestation, amniocentesis
reveals a DOD 450 of 0.20 which is at the
top of third zone of the liley curve. The
most appropriate management of such a
case is :
a) Immediate delivery
b) Intrauterine transfusion
d) Plasmapheresis
Correct Answer - B
Ans. is b i.e. Intrauterine transfusion
Liley chart
A chart that uses the spectrographic measurement of amniotic fluid
bilirubin levels plotted against gestational age to estimate the
severity of fetal hemolysis resulting from Rh isoimmunization.
The chart is divided into three zones; a measurement falling in zone
1 indicates no disease or mild disease, while one falling in zone 3
indicates severe disease with impending fetal death.
Lileys Zone I (low zone):
The fetus is unlikely to be affected
The pregnancy can be continued till term.
Lileys Zone II (mid zone):
Repeat amniocentesis by 2 weeks
Value upward -+ cordocentesis –> hematocrit <30% –> intrauterine
transfusion to increase hematocrit to 40-45%.
Preterm delivery after 34 weeks
Lileys Zone III (high zone):
The fetus is severely affected and death is imminent.
Pregnancy > 34 weeks –> delivery.
Pregnancy < 34 weeks –> cordocentesis –> hematocrit < 30% –>
intrauterine infusion to raise hematocrit to 40 - 45%.
Preterm delivery may be needed after 34 weeks.
59. The dose of anti D gamma globulin given
after term delivery for a Rh negative
mother and Rh positive baby is :
a) 50 micro gram
Correct Answer - C
300 micro gram
60. All of the following drugs are effective for
cervical ripening during pregnancy except
:
a) Prostaglandin E2
b) Oxytocin
c) Progesterone
d) Misoprostol
Correct Answer - C
Ans. is c i.e. Progesterone
61. Indication for induction of labour is :
a) Placenta previa
b) PIH at term
d) Breech
Correct Answer - B
PIH at term
62. All of the following are methods of
Induction of labour except :
a) Estrogen
b) Oxytocin
c) Stripping of membrane
d) PGE7
Correct Answer - A
Estrogen
63. Prostaglandin used for cervical ripening?
a) PG E1
b) PG E2
c) PGF2 -α
d) Oral Oxytocin
Correct Answer - B
PG E2
64. All are done in management of shoulder
dystocia except :
a) Fundal pressure
b) Mc Roberts manoeuvre
c) Suprapubic pressure
d) Woods manoeuvre
Correct Answer - A
Ans. is a i.e. Fundal pressure
The term shoulder dystocia is used to define a wide range of
difficulties encountered in the delivery of the shoulders.
Risk factors : Shoulder dystocia can occur in all those conditions
where fetus is too big or in case of mismanaged
labour.
- Fetal macrosomia°
- Maternal diabetes°
- Maternal obesity°
- Post term pregnancy°
- Anencephaly°
Fetal ascites°
H/o previous macrosomia°
- Midpelvic instrument delivery°
Management of shoulder dystocia
Shoulder dystocia should be managed as quickly as possible as
interval of time from delivery of head to delivery of body is of great
importance as far as survival of baby is concerned.
Immediately after recognition of shoulder dystocia extra help should
be called, in the form of midwifery assistance, an obstetrician, a
paediatric resuscitation team and an anaesthetist.
Maternal pushing should be discouraged, as this may lead to further
impaction of the shoulder, thereby exacerbating the situation.
Liberal episiotomy should be given to provide more space
posteriorly.
Fundal pressure should not be employed. As it is associated with an
unacceptably high neonatal complication rate and may result in
uterine rupture.
Moderate suprapubic pressure can be applied by the assistant.
Suprapubic pressure can be employed together with McRoberts'
manoeuvre to improve success rates, because suprapubic pressure
reduces the bisacromial diameter and rotates the anterior shoulder
into the oblique pelvic diameter. The shoulder is then free to slip
underneath the symphysis pubic with the aid of routine traction.
McRoberts' manoeuvre is the single most effective intervention and
should the first maneouvre to be performed. The McRobert's
manoeuvre is flexion and abduction of the maternal hips, positioning
the maternal thighs on her abdomen. It straightens the lumbo-sacral
angle, rotates the maternal pelvis cephalad and is associated with
an increase in uterine pressure and amplitude of contractions.
Aavanced manuouvfes like Vvoac s MallOGUi,r, should be used if
the McRobert's manoeuvre and suprapubic pressure fail. Woods'
corkscrew manoeuvre is progressively rotating the posterior
shoulder by 180°. So that the impacted anterior shoulder is released.
Several third line methods have been described for those cases
resistant to all simple measures. These include cleidotomy (bending
the clavicle with a finger or surgical division), symphysiotomy
(dividing the symphyseal ligament) and the Zavanelli manoeuvre.
Zavanelli manoeuvre targets at replacing the head into the pelvis
followed by cesarean section.
This manoeuvre should be the last resort as it is associated with
high degree of neonatal deaths still births and neonatal brain
damage.
Extra Edge :
Shoulder dystocia drill - Sequence to be followed in case of
shoulder dystocia.
Note: In Al-10 the same question was repeated with option a i.e.
Fundal pressure being replaced by Mauriceau smellie viet
manouvre. Rest of the options were same.
There in that question the answer would be Mauriceau Smellie viet
manoeuvre as it is not used for shoulder dystocia but for delivering
the after coming head of breech.
Second line
First line
measures Third line
measures
measures
· Call for help
· Liberal
episiotomy
· Delivery of · Cleidotomy
· Avoid fundal
posterior arm · Symphiostomy
pressure
· Wood's cork · Zavanelli
· Suprapubic
screw method manoeuvre
pressure given
· McRobert's
manoeuvre
Note: In Al-10 the same question was repeated with option a i.e.
Fundal pressure being replaced by Mauriceau smellie viet
manouvre. Rest of the options were same.
There in that question the answer would be Mauriceau Smellie viet
manoeuvre as it is not used for shoulder dystocia but for delivering
the after coming head of breech.
65. The danger of internal podalic version in
obstructed labour is :
a) Perineal tear
b) Cervical tear
Correct Answer - C
Rupture of lower uterine segment
Features Upper uterine Lower uterine segment
segment
Anatomical Upper uterine part It is formed from the uterine Dart
position between anatomical internal os and
histological internal os during labour
During :Active segment; Relatively passive segment dilate
labour contracts, retracts, and form greatly expanded , thinned
and expels the out tube through which the foetus
fetus. can pass
Thickness Becomes Becomes progressively thinned
during progressively
contraction thickened
Consistency Firm Soft
during
contraction
Peritoneal Firm Loose
attachment
Retractile Good Poor
property
Placental Left Lateral wall or Placenta previa
Placental Left Lateral wall or Placenta previa
attachment upper segment
66. The characteristics of caput succedaneum
include all of the following except :
a) Crosses midline
Correct Answer - C
It does not disappear within 2-3 days
67. The umbilical cord stump of a newborn
most frequently sloughs off about the :
a) Second day after delivery
Correct Answer - C
10th day after delivery
The cord becomes brownish black within 2 or 3 days after birth.
It falls off in about 10-14 days.
68. The effective pressure to be achieved in
vacuum extraction is :
a) 0.1 kg/cm2
b) 0.1 kg/cm2
c) 0.6 kg/cm2
d) 1.2 kg/cm2
Correct Answer - C
0.6 kg/cm2
69. Prophylactic forceps in a cardiac diseased
patient is used, when head is at station :
a) 0
b) +1
c) +2
d) -1
Correct Answer - C
+2
70. Incorrect about external cephalic version
is :
a) Done at 34 week
d) All
Correct Answer - D
All
71. Vaginal delivery is contraindicated in :
a) Central placenta praevia
b) Previous LSCS
c) Eclampsia
d) Antepartum hemorrhage
Correct Answer - A
Central placenta praevia
72. Nuchal translucency at 14 wks is
suggestive of :
a) Down's syndrome
b) Oesophageal atresia
c) Trisomy 18
Correct Answer - A
Ans. is a i.e. Down syndrome
Friends, before I go into the details of nuchal translucency I want to
first explain why I have opted for Down syndrome as the answer and
not Turner syndrome, though increased nuchal translucency is seen
in both the conditions.
My answer is based on the following lines from USG in Obs. &
Gynae. by Callen
"Johnson et al showed that simple nuchal translucency
between 10 and 14 weeks were associated with a 60% incidence
of abnormal karyotypes–mostly trisomy 21. Unlike the second
trimester experience, in which large cystic hygromas were most
often associated with turner syndrome, the 45X karyotype
represented a minority of the karyotypic abnormalities in the
group of fetuses with first trimester nuchal translucency
thickening."
Nuchal translucency and its Significance :
Nuchal translucency is a thickening of the nuchal soft tissues.
Embryos with aneuploidy have an increased amount of fluid in the
neck region resulting in increased NT measurement.
The maximum thickness of the subcutaneous translucent area
between the skin and soft tissue that overlies the fetus spine in the
sagittal plane is measured.
A cut off of 3mm is used as threshold for an abnormal nuchal
translucency in first trimester and 6mm for second trimester (16 - 20
weeks).
Best time to assess nuchal translucency in first trimester 10 - 13
weeks.
Best approach – Transvaginal (but trans abdominal is also
reasonably good).
It is best seen in the mid sagittal plane as a sonolucency at the back
of fetal neck.
Note: Increased nuchal translucency itself is not a fetal abnormality,
but rather a marker or soft sign that confers increased risk of fetal
abnormality.
Causes of Increased nuchal translucency :
Down syndrome (Trisomy 21)
Trisomy 18 Trisomy 13
Turner syndrome
Klinefelter syndrome Triploidy.
Nuchal translucency with large cystic hygroma is seen in Turners
syndrome.
Nuchal translucency with septations carries bad prognosis.
Chromosomally normal fetus with increased nuchal translucency is
associated with increased risk of :
– Cardiac abnormalities°
– Diaphragmatic hernia°
– Anterior abdominal wall defect°
– Fetal akinesia/dyskinesia syndrome°
Therefore, ACOG recommends that when nuchal translucency
measurement is 3.5 mm or more with a normal karyotype, then
targeted sonographic examination or fetal echocardiography or both
should be done.
Nuchal translucency of > 3mm in 1st trimester identifies 67% of fetus
with trisomy 21.
Increased nuchal fold thickness (> 6mm) is the most sensitive and
specific single ultrasound marker for the mid trimester detection of
down syndrome.
Extra Edge :
Nuchal translucency is the most important sonographic marker of
aneuploidy in the first trimester whereas, Nuchal fold thickness
(NFT) is the most important sonographic marker of aneuploidy in
second trimester.
NT is measured from external surface of the skull to the internal
surface of the skin.
NFT is measured from the external surface of occipital bone to the
external surface of skin.
NFT is useful for screening between 14-23 weeks.
Cut off value for NFT is 5mm (above which it is considered
abnormal).
Note: In the first trimester 2 most reliable ultrasonographic markers
for detecting Down syndrome?
1. Increased muscal translucency
2. Absent nasal bone.
73. At what level of b-HCG is it that normal
pregnancy can be earliest detected by
TVS:
a) 500 IU/m1
b) 1000 IU/m1
c) 1500 IU/m1
d) 2000 IU/m1
Correct Answer - B
Ans. is b i.e. 1000 IU/m1
PhCG level Structure
TVS/TAS
(MiU/mi) visible
• 1000 - Gestational
1200 sac TVS
Gestational
• 6000 TAS
Sac
• 7000 Yolk sac TAS
• 11000 Embryo TAS
74. What are the findings in U/S, which
suggests incompetent os :
a) Cervical length
c) Internal os
d) All
Correct Answer - D
Ans. is a, b and c i.e. Cervical length; Internal os; and Funneling of
amniotic sac
Closed cervix (competent os) on USG appears like the letter T.
Cervical incompetence :
Cervical incompetence is characterised by painless°
cervical dilatation in the second° or early third trimester° with
ballooning of the amniotic sac into the vagina°, followed by rupture
of membranes and expulsion of a usually live fetus.
It usually occurs in 16 - 24 weeks.
Diagnosis :
History : The typical history of painless rupture of membranes°
followed by the quick delivery of a live fetus in midtrimester is very
suggestive.°
Non pregnant state : Internal os allows the passage of a No. 8
Heger's cervical dilator or Foley's catheter filled with 1 ml water
without resistance.°
Premenstrual Hysterocervicography will show the typical funneling of
the internal os°.
In Pregnancy :
Transvaginal ultrasound is the ideal method to detect early
incompetence.
Incompetent os on USG shows the following features: Before
opening, the cervix shortens and then funneling can take place,
which on USG looks like the letter Y (indicating incompetent os) that
can progress to look like the letter V (cervix is just about to open).
The normal cervical length at 14 weeks is 35 - 40 mm and the
internal os diameter is less than 20 mm. A cervical length less than
30 mm and an internal os diameter more than 20 mm is suggestive
of cervical incompetence.
Other findings on ultrasound are funneling of the os.° Serial scans
may be necessary.
75. Risk of amniotic fluid embolism is
greatest in :
a) First trimester of pregnancy
c) During labour
d) In puerperal period
Correct Answer - C
During labour
76. High risk pregnancy are all except:
a) A130 incompatibility
b) Rh isoimmunisation
c) Twin pregnancy
d) Third pregnancy
Correct Answer - D
Third pregnancy
77. Use of Valproate during pregnancy may
cause :
a) Neural tube defect
b) Hydantoin syndrome
c) Respiratory depression
d) Mental retardation
Correct Answer - A
Neural tube defect
78. Best method to deliver arms in breech :
a) Lovset's method
b) Smellie veit
c) Pinard's
Correct Answer - A
Ans. is a i.e. Lovset's method
Friends amongst maipresentations - Breech is the most frequently
asked - 'Many question are asked on Breech. So, I am
summarising all the important points you need to know about
breech."
Breech
Most common cause prematurity.
Most common type of Breech : Frank Breech / Extended breech.
Incidence : – 20% at 28 weeks
– 5% at 34 weeks
– 3% at term
Commonest position – Leftsacroanterior (LSA).
Engaging diameter of breech – Bitrochanteric (10 cm).
Engaging diameter of shoulder – Bisacromial (12 cm).
Engaging diameter of head – suboccipitofrontal (10 cm).
Head is born by flexion.°
Diagnosis by Vaginal examination :
Flexed Breech (MC Extended Breech
in muitipara) Footling presentation
(MC in primigravida)
Ischia' tuberosities, Buttocks with genitalia are the • Feet are
anus, sacrum, buttocks the presenting part presenting part, feet
and feet are palpated not felt • Maximum chances of cord
and feet are palpated not felt • Maximum chances of cord
prolapse
Algorithm for
Management of Breech
Assess maternal and fetal well being
External cephalic version
Elective cesarean section (> 38 weeks)
Done after 35 completed weeks
Indications
Ideal time 36th week
• Contraindications to ECV :
– APH
- Pre eclampsia, hypertension
– Multiple pregnancy
– Obesity
– Bad obstetric history
– Elderly primigravida
– Ruptured membranes
– Oligohydramnios
– Contracted pelvis
– Congenital abnormalities of uterus
– Significant fetal anomalies/dead fetus
– IUGR
· Fetal weight < 1500 or > 3500 gm
· All complicated pregnancies eg.
–Associated with
- APH
- preeclampsia
- oligohydraminos
– Abnormal clinical pelvimetry/contracted or borderline pelvis
– Fetal distress
· If breech has :
– Hyperextended head
– Footling / Knee presentation
– Is complete breech
Previous LSCS
79. Diameter of engagement in face
presentation is :
a) 9.5 cm
b) 10 cm
c) 11.5 cm
d) 14 cm
Correct Answer - A
9.5 cm
Submento-bregmatic diameter :
The diameter extending from just behind chin to the centre of
bregma
Measures 9.5cm
Head is fully extended
Associated with face presentation
Face presentation :
Incidence : 1:600- 1:1500
Due to hyper-extension of fetal head
Presenting diameter 9.5 cm (submento-bregmatic diameter )
Engagement of fetal head late
Progression of labour slow
Probably due to lack of molding of facial bone
80. Face-to–pubis delivery often occurs in :
a) Android pelvis
b) Platypelloid pelvis
c) Anthropoid pelvis
d) Gynaecoid pelvis
Correct Answer - C
Anthropoid pelvis
81. The amniotic fluid is in balance by :
a) Excretion by fetal kidneys
b) Maternal hemostasis
d) All
Correct Answer - D
Ans. is a, b, c
Amniotic fluid originates from
In early pregnancy - As an ultrafiltrate of maternal plasma (i.e.,
option "b" correct).
By beginning of the second trimester - It consists of extracellular
fluid which diffuses through the fetal skin (i.e., option "e").
After 20 weeks - Cornification of skin prevents this diffusion and
amniotic fluid is composed of fetal urine (option "a" correct).
Also know : The water in the amniotic fluid is completely changed
and replaced in every 3 hours.
Other minor contributors : • Pulmonary fluid
Fluid filtering through the placenta
The fetus swallows 200 - 500 ml liquor every day at term.
(Option "c" correct) i.e. Excretion by fetal kidneys; Maternal
hemostasis; Fetal intestinal absorption
82. Polyhydramnios is associated with all
except :
a) Diabetes
c) Multiple pregnancy
d) Renal agenesis
Correct Answer - D
Renal agenesis
83. Maximum strain of parturient heart occurs
during:
a) At termImmediate postpartum
b) Immediate postpartum
c) 1st trimester
d) IInd trimester
Correct Answer - B
Ans. is b i.e. Immediate postpartum
"Significant hemodynamic alterations are apparent early in
pregnancy, women with severe cardiac dysfunction may experience
worsening of heart failure before mid pregnancy. In others, heart
failure develops after 28 weeks, when pregnancy induced
hypervolemia is maximal (32 weeks). In the majority, however heart
failure develops peripartum when the physiological capability for
rapid changes in cardiac out put may be overwhelmed in presence
of structural cardiac diseace "
Reading the above text, from Williams Obs.. it is clear that maximum
chances of heart failure are in the peripartum period. But it is not
clear whether maximum chances are during labour or immediate
postpartum.
Dutta Obs. 6/e, p 53 provides answer to this ?
"The cardiac output starts to increase tram 5th week of pregnancy,
reaches its peak 40-50% at about 30 - 34 weeks. Thereafter the
cardiac, output remains static till term".
"Cardiac output increases further during labour (+50%) and
immediately following delivery (+70%) over the pre labour values."
So, maximum chances of heart failure are in immediate post partum
period when cardiac output is maximum.
period when cardiac output is maximum.
Remember : Periods of maximum risk of cardiac failure :
1. Immediate postpartum
2. During delivery
3. Between 28-32 weeks (when hemodynamic changes are maximum)
4. Between 12-16 weeks of gestation (when hemodynamic changes
just begin)
5. Finally 4-5 days after delivery when charges start reverting back.
84. In a patient with heart disease, which of
the following should not be used to
control PPH :
a) Methyergometrine
b) Oxytocin
c) Misoprostol
d) Hysterectomy
Correct Answer - A
Ans. is a i.e. Methylergometrine
Antepartum Management
NYHA Class I and II :
Pregnancy and delivery are usually uneventful.
Patient can be managed on ambulatory treatment and need not be
hospitalized early.
Time of hospitalisation in Class I patients is 36 weeks° and Class II
patients is 28 weeks°.
Physical activity is limited to well within the patient's cardiac reserve.
Adequate rest is prescribed.
If any surgical procedure like tooth extraction is required, bacterial
endocarditis prophylaxis is indicated. NYHA Class III and IV :
They are at very high risk and ideally should become pregnant only
after surgical correction.
If seen in the first trimester, such patients are candidates for MTP.
If pregnancy is continued, then the women are hospitalized for the
remainder of the pregnancy.
Intrapartum Management :
This is the period of maximum risk as the cardiac output increases.
Patients should be allowed to go into spontaneous labour, if required
Patients should be allowed to go into spontaneous labour, if required
induction with vaginal PGE2 may be done (Induction is safe in case
of heart disease)
There is no place for trial of labour° in a patient with heart disease
and in such cases, elective cesarean section should be done.
Vaginal delivery is preferred unless there are obstetric indications for
cesarean section.
Only heart disease where vaginal delivery is contraindicated :
Coarctation of aorta.
Patient is laid in semi recumbent position/propped up position and if
required oxygen inhalation is given to the patient.
Pain should be adequately relieved (best done by epidural
anaesthesia)
Meticulous fluid balance (not more than 75m1/hour).
Cut short the second stage with outlet forceps.
40 mg of intravenous frusemide can be given immediately after the
baby is born°. This will divert some of the excess blood volume that
is added to the circulation by the contraction of uterus.
Management of Third stage :
Ergometrine and methergin are contraindicated.°
Oxytocin should be used in the third stage by IV infusion to reduce
the amount of bleeding. If PPH occurs in a cardiac patient—
Oxytocin, and prostaglandins can be used.
As per WHO & AGOG guidelines OXYTOCIN is the first line DOC
for atonic PPH
Rapid, continuous infusion of dilute IV oxytocin (40-80 U) in 1L NS to
be started.
Antibiotics are given as a prophylactic measure against infective
endocarditis.
Management of Puerperium
Early ambulation (to pervent thrombosis) and lactation are
encouraged.
85. Surgery for mitral stenosis during
pregnancy is ideally done at :
a) 14 weeks
b) 20 weeks
c) 28 weeks
d) 32 weeks
Correct Answer - A
14 weeks
86. Which of the following is not done for
antenatal diagnosis of Down's syndrome -
a) Amniotic fluid volume estimation
b) Alpha-fetoprotein estimation
c) Cordocentesis
Correct Answer - A
Ans is a i.e. Amniotic Fluid Volume Estimation
As discussed earlier.
In patients with previous history of Down syndrome —
"The risk of recurrence is greaser than the risk of genetic
diagnosis and these patients should be advised to seek genetic
counseling and to have a genetic diagnosis."
Since amniocentesis is not given in options therefore, chorionic
villous biopsy is the answer of choice.
Antenatal screening for Downs syndrome
Following methods are used :?
1) Triple test : It includes (i) Unconjugated estrogen (estriol) :
decreased; (ii) Maternal serum alpha-feto protein (MSAFP):
decreased; and (iii) hCG : increased (Note : All these three markers
are decreased in Edward syndrome)
2) New markers : These are (i) Increased inhibin A in maternal
blood; and (ii) Decreased PAPA (pregnancy associated plasma
protein).
3) USG : It shows : (i) Increased nuchal translucency (increased
nuchal fold thickness) in first trimester; (ii) Ductus venous flow
reversed; and (iii) Nasal bone hypoplasia.
Karyotyping : It can be done by chorionic villus sampling at 10-12
weeks or aminocentesis
87. Best parameter for ultrasound evaluation
of IUGR is
a) Placental membrane
b) Length of femur
c) Abdominal circumference
d) BPD
Correct Answer - C
Ans. is c i.e. Abdominal circumference
"Abdominal circumfernece (AC) is the single most sensitive
parameter to detect IUGR. Serial measurements of AC and
estimations of fetal weight are more diagnostic to fetal growth
restriction."
88. All of the following indicate Fetal distress
except:
a) Thick (pea-soup) meconium
Correct Answer - D
Fetal skull blood pH>7.32
89. MC cause of secondary amenorrhoea is:
March 2004
a) Pregnancy
b) TB
c) Thyrotoxicosis
Correct Answer - A
Ans. A i.e. Pregnancy
90. Triple test for Down syndrome is by using
all EXCEPT:
September 2012
a) Maternal HCG
b) Maternal Estriol
c) Maternal Alphafetoprotein
d) Maternal Inhibin A
Correct Answer - D
Ans. D i.e. Maternal Inhibin A
Inhibin A is counted in Quadruple test for Down syndrome.
91. Weight of placenta at term:
March 2013
a) 300 grams
b) 400 grams
c) 500 grams
d) 600 grams
Correct Answer - C
Ans. C i.e. 500 grams
Placenta is a circular disc with diameter of 15 - 20 cm.
Feels spongy and weighs 500 gm.
At term, the weight of placenta to fetus is about 1 : 6
At term, 4/5th of the placenta is of the fetal origin, i.e., majority of
placenta is of fetal and not maternal origin.
Maternal portion amounts to less than 1/5th of the placenta.
Placenta is nothing but a specialized part of the chorion.
92. Weight gain during pregnancy:
March 2013
a) 4-6 Kg
b) 7-9 Kg
c) 10-12 Kg
d) 13-15 Kg
Correct Answer - C
Ans. C i.e. 10-12 Kg
93. Which of the following is correct for
urinary system changes in pregnancy:
March 2013
a) Renal blood flow decreased
c) GFR is increased
d) Kidneys shrink by 1 cm
Correct Answer - C
Ans. C i.e. GFR is increased
94. Most common type of vertex presentation:
March 2005
a) Right Occipito Posterior
Correct Answer - D
Ans. D: Left Occipito Anterior
Fetal presentation:
Cephalic (head-first) presentation:
Cephalic presentation is considered normal and occurs in about
96.5% of deliveries. There are different types of cephalic
presentation, which depend on the fetal attitude.
Vertex occupying the left anterior quadrant of the pelvis is the
commonest position (13%) and is known as L.O.A. (left occipito
anterior) though L.O.T. (left occipito transverse) is commonest
accounting for 40% of vertex presentations.
95. Polyhydroaminosis is volume of amniotic
fluid more than:
September 2005
a) 1000 ml
b) 2000 ml
c) 3000 ml
d) 4000 ml
Correct Answer - B
Ans. B: 2000 ml
Because the normal values for amniotic fluid volume increase during
pregnancy, the actual volume that constitutes polyhydramnios is
dependent on the gestational age of the fetus.
Polyhydramnios usually refers to amniotic fluid volumes greater than
2000 ml.
The range of fluid values diagnostic of oligohydramnios is not as
wide as that for polyhydramnios. Less than 200 ml, or when amniotic
fluid index is less than 5 cm, is usually considered to be indicative of
oligohydramnios.
96. Maximal level of cardiac output is seen at:
September 2009
a) 22-26 weeks of pregnancy
Correct Answer - C
Ans. C: 30-34 Weeks of Pregnancy
The cardiac output starts to increase from 5th week of pregnancy
and reaches its peak 40-50% at about 30-34 weeks. Cardiac output
increase further during labour (+50%) and immediately following
delivery (+70%) It returns to pre-pregnancy level by 4 weeks time
after labour.
97. pH of vagina in pregnancy is:
September 2009
a) 2-4
b) 4- 6
c) 6-8
d) 8-10
Correct Answer - B
Ans. B: 4- 6
Vaginal walls becomes hypertrophied and more vascular.Increased
blood supply of the venous plexus surrounding the walls gives the
bluish colouration of the mucosa (Jacquemier's sign)
Vaginal secretion becomes copious, thin and curdy white, due to
marked exfoliated cells and bacteria. The pH becomes acidic (3.5-6)
due to more conversion of glycogen into lactic acid by the
lactobacillus.
98. Best time for diagnosing fetal
abnormalities by USG:
September 2009
a) 6-12 weeks of pregnancy
Correct Answer - B
Ans. B: 13-19 weeks of pregnancy
99. Increase in cardiac output seen in
pregnancy is:
March 2010
a) 10%
b) 20%
c) 30%
d) 40%
Correct Answer - D
Ans. D: 40%
The cardiac output starts to increase from 5t1 week of pregnancy
and reaches its peak 40-50% at about 30-34 weeks. Cardiac output
increase further during labour (+50%) and immediately following
delivery (+70%)
100. During application of the cup in
Ventouse, 'knob' of the cup points
towards:
March 2013
a) Brow
b) Chin
c) Neck
d) Occiput
Correct Answer - D
Ans. D i.e. Occiput
Ventouse Delivery:
The suction cup should be placed symmetrically astride the saggital
suture at the median flexion point (pivot point) which is 2 cm onturtor
to the posterior fontanelle or 6 cm posterior to anterior fontanelle
Indications
As an alternative to forceps operation.
Deep transverse arrest with adequate pelvis
Delay in descent of head of the second baby of twins
Delay in first stage due to uterine inertia or primary cervical dystocia
Contraindications
Prematurity
Head not engaged
Fetal distress
Pelvic contraction
Transverse lie
After coming head of breech
Partially dilated cervix
Congenital anomalies
Dead fetus.
Complications
Cephalhematoma
Subaponeurotic or subgaleal haemorrhage
Chignon
Retinal hemorrhage
101. Partogram is used for:
March 2011
a) Determining pattern of fetal heart rate
Correct Answer - D
Ans. D: Recording progress of labour
Partograph is a composite graphical record of key data (maternal
and fetal) during labour, entered against time on a single sheet of
paper
Remember:
Assessment of labour is best done by partogram
Graph showing relationship between cervical dilatation and labour is
cervicograph
Bishop's score includes:
Position of cervix
Consistence of cervix
Dilatation of cervix
Effacement of cervix
Station of head
102. Inversion of the uterus, a complication is
noticed in:
September 2011
a) 1st satge of labour
Correct Answer - C
Ans. C: IIIrd stage of labour
Inversion of the uterus is a rare but a life threatening complication in
IIIrd stage in which uterus is turned inside out partially or completely
Uterine inversion:
Inside out turning of uterus
Results from mismanaged 3rd stage of labour
MC complication associated with inversion: Haemorrhage
103. Anti-D Rh is given for:
September 2005
a) Rh positive father,Rh positive mother
Correct Answer - D
Ans. D: Rh positive father,Rh negative mother
Rho(D) Immune Globulin is given by intramuscular injection that is
used to prevent the immunological condition known as Rhesus
disease (or hemolytic disease of newborn) particularly when father is
Rh positive and mother is Rh negative.
It is a solution of IgG anti-D (anti-RhD) antibodies that bind to, and
lead to the destruction of, fetal Rh D positive red blood cells that
have passed from the fetal circulation to the maternal circulation.
Therefore, in a Rhesus negative mother it can prevent sensitization
of the maternal immune system to Rh D antigens, which can cause
rhesus disease in the current or in subsequent pregnancies
104. True about HCG:
September 2012
a) Produced by cytotrophoblast
Correct Answer - B
Ans. B i.e. Prevent involution of corpus luteum
Beta-HCG
Human Chorionic Gonadotropin
It is produced by the syncytiotrophoblast of the placenta and
secreted into blood of both mother and fetus.
It can be detected in maternal serum or urine as early as 1- 9 days
of feritisation
The blood and the urine values reach maximum levels ranging
between 100 IU - 200 IU /ml between 60 - 70 days of pregnancy.
The concentration falls slowly to I0 - 20 IU/ml by 100 - 130 days of
pregnancy.
It is chemically and functionally similar to pituitary leutenising
hormone
It acts as a stimulus for the secretion of progesterone by the corpus
luteum of pregnancy.
The rescue and maintenance of corpus luteum till 6 weeks of
pregnancy is the major biological function of hCG.
It stimulates the leydig cells of the male fetus to produce
testosterone. t It has immunosuppressive activity.
It stimulates adrenal and placental steroidogenesis and has
thyrotropic activity.
It promotes secretion of relaxin from corpus luteum.
High levels of hCG are detected in multiple pregnancy, hydatidiform
mole, choriocarcinoma and down's syndrome.
Low levels of hCG are found in ectopic pregnancy and spontaneous
abortion.
105. Most common site of ectopic pregnancy
in fallopian tube:
March 2005
a) Infundibulum
b) Ampulla
c) Isthmus
d) Interstitium
Correct Answer - B
Ans. B: Ampulla
The vast majority of ectopic pregnancies implant in the Fallopian
tube.
Pregnancies can grow in the infundibulum (18% of all ectopics), the
ampullary section (55%), the isthmus (25%), and the interstitial part
of the tube (20%).
Ampullary pregnancy ruptures generally at 8 weeks
Mortality of a tubal pregnancy at the isthmus or within the uterus
(interstitial pregnancy) is higher as there is increased vascularity that
may result more likely in sudden major internal hemorrhage.
106. A 20-year old average weight female
complains of oligomenorrhea along with
facial hair. Preliminary investigations
reveal raised free testosterone levels.
USG Pelvis: ovary shows normal
morphology. Which of the following
could be likely etiology
a) Idiopathic hirsutism
b) PCOD
c) Adrenal hyperplasia
Correct Answer - B
Ans. b. PCOD (Ref Jeffcott 6/e p205; Shaw's 14/331-332, 13/e
p353-354; Novak's 14/1077, 1082)
107. A girl with normal stature and minimal or
absent pubertal development is seen in
a) Kallman syndrome
b) Turner syndrome
Correct Answer - A
Ans. a. Kallman syndrome
'Kallmann syndrome is a genetic condition where the primary
symptom is a failure to start puberty or a failure to fully
complete it. It occurs in both males and females and has the
additional symptoms o.f hypogonadism and almost invariably
infertility. Kallmann syndrome al.so features the additional symptom
of an altered sense of smell; either totally absent
(anosmia) or highly reduced (hyposmia)- Nelson
108. A child with nephrotic syndrome
following an episode of diarrhea
presented with acute kidney injury with a
creatinine of 4.5. All of the following are
possible reasons except?
a) Excess furosemide
Correct Answer - D
Ans. d. Steroid induced diabetes
Massive proteinuria, with the daily loss of 3.5 gm or more of protein
Hypoalbuminemia, with piasma albumin levels < 3 gm/dl
Generalized edema
Hyperlipidemia and lipiduria
Steroid induced diabetes is not the csuse of acute kidney injury in
patients of nephrotic syndrome who present with episode
of diamhea and raised creatinine.
109. Which of the following displacement is
not seen in Colle's fracture?
a) Radial tilt
b) Volar tilt
c) Dorsal displacement
d) Supination
Correct Answer - B
Ans. b. Volar tilt
Displacements of Colle's Fracture:
It results in dinner fork / silver fork / spoon shaped deformity
110. In threatened abortion, the size of uterus:
a) Smaller than expected
Correct Answer - C
Ans. C.;Corresponds to duration of amenorrhea
SYMPTOMS:
Bleeding may be only slight spotting, or it can be heavy.
Pain and cramping are in the lower abdomen. They may be on one
side, both sides, or in the middle. The pain can go into the lower
back, buttocks, and genitals.
111.
Which of the following is not a part of HELLP
syndrome?
a) Hemolysis
c) Thrombocytopenia
d) Retroplacental hemorrhage
Correct Answer - D
Ans. d. Retroplacental hemorrhage
Retroplacental hemorrhage is not a part of HELLP syndrome.
HELLP syndronte is an acronymfor Hemolysis (H), Elevated liver
enzymes (EL) and Low Platelet count (LP) of < 1,00,000/mm3
Clinical Features:
. Manifested by nausea, vomiting, epigastric or right upper quadrant
pain, along with biochemical and
haematological changes.
. Parenchymal necrosis of liver iauses elevation in hepatic enzymes
(AST and ALT > 70 IU/L and LDH > 600 IU/L).
. There may be subcapsular hematoma formation (which may need
CT scanning) and abnormal peripheral smear.
. Eventually liver mav rupture to cause sudden hypotension, due to
hemoperitoneum
112. Which of the following is responsible for
pubertal growth in females?
a) Decreased level of adrenal androgens at puberty
Correct Answer - C
Ans. c. Pulsatile release of GnRH during sleep
uAfier a decade of quiescence, pakatile secretion of GnRH
increases and the hTpothalamic-pitaitary gonadal axis is reactivated
(gonadarche), probably in response to metabolic signals from the
periphery. FSH and LH levels rise moderutely before age 10,
foltowed by a gradaal increase in estradiol concentrdtions, which
stimulate breast development (thelarehe). The incrcase in
pulsatile gonadotrophin secretion occurs first at night, during sleep,
but gradually extends throughout the day
113. Which of these is diagnostic of
menopause?
a) Serum FSH > 40
b) Serum LH > 20
Correct Answer - A
Ans: A. Serum FSH > 40
(Ref. Shaw’s. 16/e p66. 15/e p62).
Serum FSH > 40 1U/L is diagnostic of menopause.
Criteria for Menopause
Estrogen (E2): 10-20 pg/ml° E2/E1 < 1°
Estrone (El): 30-70 pg/ml° Urine FSH > 40 IU/L°
Laboratory diagnosis:
FSH & estrogen level - Assess ovarian failure.
Especially in premature ovarian failure case or women seeking
treatment for infertility.
FSH levels:
Greater FSH level (>40 m IU/ml).
Documents ovarian failure associated with menopause.
Estrogen level:
Normal/elevated - Depending on stage of menopausal transition.
After menopause extremely low estrogen level.
Evaluated to assess women's response to hormone replacement
therapy.
114. A 30 years old G3P2 with 10 weeks of
amenorrhea comes with an intrauterine
pregnancy with intra uterine
contraceptive device in situ. On pelvic
examination, the string of the IUCD was
visible at the cervical os. Patient wishes
to continue pregnancy. What will you
do?
a) Leave IUCD and continue pregnancy
Correct Answer - D
Ans: D. Remove intrauterine contraceptive device
(Ref: Dutta 8Ie p618-619, 6Ie p540)
Women who become pregnant with an IUCD in situ should be
informed of the increased risks of second-trimester miscarriage,
preterm delivery and infection if the intrauterine method is left in situ.
Removal would reduce adverse outcomes but is associated with a
small risk of miscarriage.
115. Which is the most common congenital
abnormality in a baby of a diabetic
woman?
a) Ventricular septal defect
b) Anencephaly
c) Meningomyelocele
d) Sacral agenesis
Correct Answer - A
Ans: A Ventricular septal defect
Explanation:
(Ref. Nelson 20/e p898)
Most common congenital abnormality in a baby of diabetic women is
ventricular septa! defect.
Congenital anomalies are increased threefold in infants of diabetic
mothers.
Most common anomalies:
Cardiac malformations (ventricular or atrial septal defect,
transposition of the great vessels, truncus arteriosus, double-outlet
right ventricle, tricuspid atresia, coarciation of the aorta).
Lumbosacral agenesis.
Additional anomalies:
Neural tube defects, hydronephrosis, renal agenesis and dysplasia,
duodenal or anorectal atresia, situs in versus, double ureter, and
holoprosencephaly.
These infants may also demonstrate abdominal distention caused by
a transient delay in development of the left side of the colon, the
small left colon syndrome.
116. According to WHO guidelines, which of
the following is true about management
of second stage of labor?
a) Manual support of perineum to maintain continuous deflexion of
head
Correct Answer - A
Answer- A. Manual support of perineum to maintain continuous
deflexion of head
Do not perform perineal massage in 2nd stage of labor.
Either the hands on'(guarding the perineum & deflexing the baby's
head) or the 'hands poised' (with hands off the perineum and baby's
head but in readiness) technique can be used to facilitate
spontaneous birth.
Do not offer lidocaine spray to reduce pain in 2nd stage oflabor.
Do not carry out a routine episiotomy during spontaneous vaginal
birth.
Do not offer episiotomy routinely at vaginal birth after previous third-
or fourth-degree trauma
117. A lady presented with 7 weeks
amenorrhea presented with slight
vaginal spotting. CRL was 5mm with
well-formed gestational sac with
calculated GA of 5.6 weeks on TVS. Next
line of management?
a) Wait for another I week and repeat TVS
Correct Answer - A
Ans: A. Wait for another I week and repeat TVS
Ref: Williams ohstetrics, 24'h ed.
An intrauterine gestational sac is reliably visualized with transvaginal
sonography by 5 weeks, and an embryo with cardiac activity by 6
weeks.
The embryo should be visible transvaginally once the mean sac
diameter has reached 20 mm, otherwise the gestation is
anembryonic.
Cardiac motion is usually visible with transvaginal imaging when the
embryo length has reached 5 mm. If an embryo less than 7 mm is
not identified to have cardiac activity, a subsequent examination is
recommended in 1 week.
118. During progression of normal
pregnancy, S/D ratio in umblical artery ?
a) Decreases
b) Increases
c) Plateau
d) Not significant
Correct Answer - A
Ans. is 'a' i.e., Decreases
The commonly used parameters in the umbilical artery doppler
are:
SD ratio: systolic velocity/diastolic velocity
Pulsatility index (PI) (Gosling index): (PSV - EDV)/ TAV
Resistive index (RI) (Pourcelot index): (PSV - EDV) / PSV
Normally the parameters mentioned above should decrease
progressively as the pregnancy advances, as there is increase in the
end diastolic velocity due to growth and dilatation of umbilical
circulation.
Any deviation in the parameters is suggestive of either IUGR [weak
fetal heart pump thus progressive decrease flow in the diastolic
phase] or pre - eclampsia [high pressures in the downstream
vessels progressively decrease the velocity of flow in the umbilical
artery during fetal diastole].
Thus, during progressive monitoring of the umbilical artery
Doppler, as the severity of the disease increases, we observe
first:
The decreased velocity in the end diastolic phase
Then absent flow in the end diastolic phase followed by
Reversal of flow in the end diastolic phase.
119. A pregnant lady has delivered a baby 35
minutes back. However, placenta has not
been delivered. What should be the next
line of management.
a) Controlled cord traction
Correct Answer - B
Ans. is 'b' i.e., Manual removal of placenta
Manual Extraction:
If the placenta has not delivered after 30 minutes, or if separation
has occurred, without delivery of the placenta, manual removal may
be performed to reduce excessive blood loss. Injection of oxytocin
into the umbilical vein is a safe procedure and may prevent the need
for manual removal of the placenta in some women. Intrauterine
bacterial contamination is a theoretical risk of manual extraction, but
is not a common complication.
120.
Elective caesarion section in HIV infected
pregnant woman is done in ?
a) All cases
b) All primipara
Correct Answer - C
Ans. is 'c' i.e., If viral load > 1000 /ml
A scheduled cesarean delivery (sometimes called a C-section) can
reduce the risk of mother-to-child transmission of HIV in women who
have a high viral load (more than 1,000 copies/mL) or an
unknown viral load near the time of delivery. A cesarean delivery
to reduce the risk of mother-to-child transmission of HIV is
scheduled for the 38th week of pregnancy, 2 weeks before a
woman’s expected due date.
121. A 38 weaks pregnant lady delivered baby
without upper limb. What can be the
cause ?
a) Amniotic band
c) Genetic abnormality
d) None
Correct Answer - A
Ans. is 'a' i.e., Amniotic band
It is a case of limb reduction defect or congenital amputation in
which there is defect in formation of a part of limb or the entire limb.
The exact cause of congenital amputation is unknown and can result
from a number of causes. However, most cases show that the first
three months in a pregnancy are when most birth defects occur
because that is when the organs of the fetus are beginning to form.
One common cause is amniotic band syndrome, which occurs when
the inner fetal membrane (amnion) ruptures without injury to the
outer membrane (chorion).
Fibrous bands from the ruptured amnion float in the amniotic fluid
and can get entangled with the fetus, thus reducing blood supply to
the developing limbs to such an extent that the limbs can become
strangulated, the tissues die and are absorbed into the amniotic
fluid.
122. Viability of fetus is beyond ?
a) 25 weeks
b) 28 weeks
c) 30 weeks
d) 32 weeks
Correct Answer - B
Ans. is 'b' i.e.,28 weeks
Viability means the physical ability of a fetus to lead a separate
existence after birth apart from its mother, by virtue of a certain
degree of development. A child is viable after 210 days (7 months)
of intrauterine life, and in some cases after 180 days (6 months) but
in most of these cases fetus is immature.
123. A 32-year-old primigravida at 39 weeks of
gestational age has a blood pressure
reading of 150/100 mm Hg obtained
during a routine visit. Her baseline blood
pressure during the pregnancy was
120/70 mmHg. The patient denies any
headache, visual changes, nausea,
vomiting, or abdominal pain. Her repeat
BP is 160/90 mmHg, and urinalysis is
negative for protein. Which of the
following is the most likely diagnosis?
a) Preeclampsia
c) Eclampsia
d) Gestational hypertension
Correct Answer - D
Ans. is 'd' i.e.,Gestational hypertension
Hypertension in pregnancy is defined as blood pressure of 140/90
mmHg or greater on at least two separate occasions that are 6 h or
more apart.
The presence of edema is no longer used as a diagnostic criterion
because it is so prevalent in normal pregnant women.
A rise in systolic blood pressure of 30 mmHg and a rise in diastolic
blood pressure of 15 mmHg are also no longer used.
In gestational hypertension, maternal blood pressure reaches
140/90 or greater for the first time during pregnancy, and proteinuria
is not present.
In preeclampsia, blood pressure increases to 140/90 after 20 weeks
of gestation and proteinuria is present (300 mg in 24 h or 1+ protein
or greater on dipstick). Eclampsia is present when women with
preeclampsia develop seizures.
124. End point of D/C is ?
a) After 2 strokes
c) Uterine cry
Correct Answer - C
Ans. is 'c' i.e., Uterine cry
The curette should be inserted gently to the uterine fundus and
withdrawn slightly. To decrease the risk of perforation, do not force
the curette.
The end point of this scrubbing should be the detection of a
scratching/grating sensation or sound (the "uterine cry"), which
represents a sharp curette running over myometrium. Too vigorous
a pursuit of this end point may lead to formation of synechiae
(Asherman's syndrome).
125. The fetus (which is partly foreign to the
mother) is not rejected by mother due to
:
a) Immunosuppressive effect of placental hormones
Correct Answer - D
Ans. is 'd' i.e., All of the above
The fetus and the placenta contain paternally determined
antigens which can lead to immunological rejection. The
placenta has some role in preventing such a rejection:
Placental hormones (EPF,PAPP-A,STERIODS & HCG) have got
some immunosuppressive effect.
There is production of blocking antibodies by mother in response to
TLX (trophoblast lymphocyte cross reactive antigen) which protect
the fetus from rejection.
Absence of HLA molecules in villous trophoblast.
126. The production rate of the following
hormone near term, is the greatest of any
known hormone in humans
(approximately lgm/day)-
a) Relaxin
b) Progesterone
c) hCG
d) hPL
Correct Answer - D
Ans. is 'd' i.e., hPL
Human placenta lactogen (hPL) was named so , because of its
potent lactogenic & growth hormone like bioactivity, as well as
immunochemical resemblance to human growth hormone.
It is detected as early as 2nd or 3rd week after fertilization.
Maternal plasma concentrations are linked to placental mass & they
rise steadily till 34-36 weeks of gestation.
The hPLproduction rate near term is by far the greatest of any
known hormone in humans- approximately 1gm/day.
127. Incidence of infertility isin reproductive
age couples ?
a) 0-5 %
b) 5-10%
c) 10-20 %
d) 25-30%
Correct Answer - C
Ans: C. 10-20 %
Infertility is defined as the failure to conceive after one year of
regular unprotected intercourse.
Incidence
10-20% of reproductive ages couples.
128. The most important risk factor for
development of post partum uterine
infection/sepsis is:
a) Anemia
b) Route of delivery
c) Diabetes
d) Obesity
Correct Answer - B
Ans. is 'b' i.e., Route of delivery
The route of delivery is the single most significant risk factor for the
development of uterine infection.
Compared with cesarean delivery, metritis following vaginal delivery
is relatively uncommon.
Most female pelvic infections are caused by bacteria indigenous to
the female genital tract.
129. Maturation index in mid-secretory phase
of menstrual cycle is ?
a) 0/95/5
b) 80/20/0
c) 0/70/30
d) 0/95/5
Correct Answer - C
Ans. is 'c' i.e., 0/70/30
Maturation index (MI) is the relative percentage of parabasal,
intermediate and superficial cells per 100 cells counted.
MI is expressed in 3 numbers—the left one parabasal percentage,
intermediate in the center and on the right, the percentage of
superficial cells.
At birth 0/95/5
Childhood 80/20/0
Preovulatory 0/40/60
Mid-secretory 0/70/30
Pregnancy 0/95/5
Postpartum 100/0/0
Postmenopause0/100/0 or 100/0/0
130. MC cause of anovulation is?
a) PCOS
b) Hyperprolactinemia
Correct Answer - A
Ans. is 'a' i.e.,PCOS
Anovulation and ovulatory dysfunction can be caused by a number
of factors.
The most common cause of ovulatory dysfunction is polycystic
ovarian syndrome, or PCOS.
Other potential causes of irregular or absent ovulation:
Obesity
Underweight
Extreme exercise
Hyperprolactinemia
Premature ovarian failure
Perimenopause, or low ovarian reserves
Thyroid dysfunction
Extremely high levels of stress
131. Contraindications for medical method
(mifepristone misoprostol) of first
trimester MTP?
a) Patient with hemoglobin of 7 gm%
c) Glaucoma
Correct Answer - D
Ans. is 'd' i.e., All of the above
Contraindications (due to medical reasons) for medical method
of abortion:
Smoking > 35 years
Hemoglobin < 8 g%
Ectopic pregnancy/ adnexal mass
Coagulopathy
Chronic adrenal failure
Uncontrolled hypertension ( BP>160/100mmHg)
CVD
Severe renal, hepatic or respiratory diseases
Glaucoma
Uncontrolled seizure
Allergy or intolerance to mifepristone /misoprostol or other
prostaglandins
Lack of access to 24 hours emergency services
132. Cervical stroma consists of?
a) Racemose glands
b) Tubular glands
c) Alveolar glands
Correct Answer - A
Ans. is 'a' i.e., Racemose glands
The glands which dip into the stroma are of complex racemose type
and are lined by secretory columnar epithelium.
There is no stroma in the cervix unlike the corpus and the lining
epithelium rests on a thin basement membrane.
The change in the epithelium and the glands during menstrual cycle
and pregnancy are not so much as those in the endometrium.
133. Human placenta is ?
a) Discoid
b) Hemochorial
c) Deciduate
Correct Answer - D
Ans. is 'd' i.e., All the above
The human placenta is :
Discoid, because of its shape.
Hemochorial, because of direct contact of the chorion with the
maternal blood andDeciduate, because some maternal tissue is
shed at parturition.
134. In which type of abortion the gestational
age corresponds to the uterine size?
a) Threatened
b) Inevitable
c) Complete
d) Mixed
Correct Answer - A
Ans. A.Threatened
In Threatened abortion the gestational age corresponds to the
uterine size.
135. Congestive dysmenorrhea is seen in
patients with?
a) Fibroid
b) IUD wearers
c) PID
Correct Answer - D
Ans. is 'd' i.e., All the above
Dysmenorrhoea is described under three clinical varieties
Spasmodic dysmenorrhoeais the most prevalent and manifests as
cramping pains, generally most pro- nounced on the first and second
day of menstruation.
Congestive dysmenorrhoeamanifests as increasing pelvic discomfort
and pelvic pain a few days before menses begin. Thereafter, the
patient rapidly experiences relief in the symptoms. This variety is
commonly seen in PID, IUCD wearers, pelvic endometriosis and
fibroids. It is also experienced by women having varicosity of pelvic
veins.
Membranous dysmenorrhoeais a special group in which the
endometrium is shed as a cast at the time of men- struation. The
passage of the cast is accompanied by painful uterine cramps. This
is a rare variety.
136. Intracytoplasmic injection of sperm is
done when the infertile male has sperm
count less than?
a) 3 million/ml
b) 4 million/ml
c) 5 million/ml
d) 6 million/ml
Correct Answer - C
Ans. is 'c' i.e., 5 million/ml
Indications of ICSI in male infertility comprise:
Sperm count less than 5 million/mL.
Decreased or absent motility of sperms.
Many abnormal sperms.
Previous failed IVF.
Unexplained infertility.
137. Pregnancy as a result of contraceptive
failure can be terminated as per which
indication of the MTP act ?
a) To save life of mother
b) Social indication
c) Eugenic indication
Correct Answer - B
Ans. is 'b' i.e., Social indication
Social indications
1. This is almost the sole indication and is covered under the provision
"to prevent grave injury to the physical and mental health of the
pregnant woman".
2. In about 80%, it is limited to parous women having unplanned
pregnancy with low socioeconomic status.
3. Pregnancy caused by rape or unwanted pregnancy caused due to
failure of any contraceptive device also falls in this category (20%).
138. Androgenic activity of the 19
nortestosterone nucleus is decreased by
?
a) Adding alkyl group at C 17
Correct Answer - B
Ans. is 'b' i.e., Removal of methyl group at C 19
Substitution of a 7α-methyl group in 17β-hydroxyestr-4-en-3-one
(19-nortestosterone) or its 17α-methyl derivative enhanced the
androgenic activities of the parent compounds as well as their ability
to bind to β protein and to prostate cell nuclei. Thus, 7α, 17α-
dimethyl-19-nortestosterone had an apparent affinity for βprotein
several times higher than that of 5α-dihydrotestosterone.
139. Hypogonadotropic primary amenorrhoea
is treated by?
a) Gonadotropin therapy
Correct Answer - A
Ans. is 'a' i.e., Gonadotropin therapy
Treatment of Hypogonadotropic Primary Amenorrhoea:
These women have FSH level less than 40 mIU/mL.
Hypogonadotropinaemia leading to hypogonadism is usually the
result of hypothalamic dysfunction, pituitary failure or systemic
illnesses.
Administration of GnRH helps to differentiate hypothalamic
dysfunction from pituitary failure. In patients with pituitary failure
systemic LH level will not rise after giving GnRH.
Empty sella turcica, cause of pituitary failure, is characterized by
herniation of subarachnoid membrane into the pituitary sella turcica.
Absence of pituitary gland causes absence or low level of FSH and
LH.
In such patients gonadotropin hormone therapy is required for
management of primary amenorrhoea.
140. Nugent's scorefor bacterial vaginosis
includes all except?
a) Lactobacillus
b) Gardnerellavaginalis
c) Mobiluncus
d) Gonococcus
Correct Answer - D
Ans. is 'd' i.e., Gonococcus
Nugent’s score for diagnosis of bacterial vaginosis (BV)
The Nugent Score is a system employed for diagnosing BV using
microscopic examination of a Gram-stained smear of vaginal
discharge.
It is used primarily in research studies rather than clinical practice.
Scores are calculated by assessing predominance of three types of
bacteria morphology and staining:
1. Large gram-positive rods (Lactobacillus spp.) decrease in
lactobacillus scored as 0 - 4,
2. Small gram-variable rods (G vaginalis or Bacteroides spp.) scored
as 0 - 4, and
3. Curved gram-variable rods (Mobiluncusspp.) scored as 0 - 2.
A score of 7 to 10 is consistent with bacterial vaginosis.
A score of 0-10 is generated from combining three other
scores. The scores are as follows:
0–3 is considered negative for BV
4–6 is considered intermediate
7+ is considered indicative of BV.
Curved
Gram
variable
variable
rods —
average
per high
powered
Lactobacillus
(1000× oil
morphotypes —
immersion)
average per high Gardnerella / Bacteroides morphotypes
field.
powered — average per high powered
View
(1000× oil (1000× oil immersion) field. View
multiple
immersion) field. multiple fields.
fields (note
View multiple
that this
fields.
factor is
less
important
— scores
of only 0–2
are
possible)
Score 0 for >30
Score 1 for 15–
30
Score 0 for
Score 2 for >= 1- Score 0 for 0
0
14 Score 1 for0, yet <1)
Score 1 for
Score 3 for < 1 Score 2 for 1–4
< 5
(this is an Score 3 for 5–30
Score 2 for
average, so Score 4 for > 30
>= 5
results can be
>0, yet <1)
Score 4 for 0
141. 30 years old female presents with
chronic cyclical abdominal pain. Pain is
increased during the 5 days of menstrual
cycle. Patient is married for 2 years and
has not conceived till now. What is the
most appropriate next step in
management?
a) Ultrasonography
Correct Answer - A
Ans. is 'a' i.e., Ultrasonography
According the clinical scenario of chronic cyclical abdominal pain
with infertility the most probable clinical diagnosis is endometriosis
and the most logical next step in management of this patient is to
investigate the patient with ultrasonography for definitive diagnosis
of endometriosis.
142. Which of the following is the most
suitable management for a patient who is
known case of Glucose 6 phosphate
Dehydrogenase deficiency and presents
with 6 months of amenorrhea and Hb of
6gm%?
a) Immediate blood transfusion
d) Both b and c
Correct Answer - D
Ans. is d i.e., Both b and c
The patient in question has severe anemia (
143. Which of the following is true regarding
breast feeding in HIV positive mother?
a) Breast feeding has no effect on mother to child transmission of
infection
Correct Answer - B
Ans. is 'b' i.e., Breast feeding doubles the risk of mother to
child transmission of infection
Breastfeeding doubles the risk of maternal to child transmission
(14% to 28%) of infection.
144. Following are the risk factors for pelvic
organ prolapse except ?
a) Vaginal child birth
b) Hypoestrogenism
c) Constipation
d) Crohns disease
Correct Answer - D
Ans. is 'd' i.e., Crohns disease
Risk factors associated with pelvic organ prolapse are :
1. Pregnancy and vaginal delivery
2. Menopause (Aging, hypoestrogenism)
3. Increased intra-abdominal pressure (COPD, constipation, obesity)
4. Pelvic floor trauma
5. Genetic factors (Race, connective tissue disorders)
6. Spina bifida
145. Which enzyme deficiency is most
commonly responsible for presence of
long clitoris and fused vagina?
a) 21 hydroxylase
b) 11 hydroxylase
Correct Answer - A
Ans. is 'a' i.e., 21 hydroxylase
Features of long clitoris and fused vagina are suggestive of
adrenogenital syndrome/ congenital adrenal hyperplasia (CAH). 21
hydroxylase deficiency is responsible for 95% cases of CAH.
146. Which of the following is false about
elongation of cervix ?
a) The supravaginal portion is stretched and elongated
Correct Answer - C
Ans. is 'c' i.e., It is uncommon for the gland to elongate beyond
5cms
Elongation of the Cervix
In this condition the supravaginal portion of the cervix is well
supported by Mackenrodt ligaments but the vaginal portion of the
cervix prolapses with the vagina so the supravaginal portion gets
stretched and elongated.
This usually happens with second degree and third degree prolapse
of the uterus.
With procidentia, the entire uterus slides with the vagina and hence
the cervix retains its normal length.
It is not uncommon for the cervix to elongate to as much as 10 cm in
length.
The cervix may show hypertrophy and congestion.
The uterus is invariably retroverted.
147. Teratogens produce all or none effect till
which day of gestation?
a) 21
b) 31
c) 41
d) 51
Correct Answer - B
Ans. is 'b' i.e., 31
Timing of Teratogen exposure and The hazards
Before D 31: Teratogen produces an all or none effect. The
conceptus either does not survive or survives without anomalies. In
early conception only few cells are there. So any damage at that
phase is irreparable and is lethal.
D 31-D 71 is the critical period for organ formation. Effects of
teratogen depend on the following factors: (i) Amount of the drug
reaching the fetus, (ii) Gestational age at the time of exposure, (iii)
Duration of exposure.
After D 71 development of other organs continues. Diethylstilbestrol
(DES) related uterine anomalies occur with exposure around 20
weeks.
Brain continues to develop throughout pregnancy and neonatal
period. Fetal alcohol syndrome occurs in late pregnancy.
148. Staging laprotomy is usually done for ?
a) Carcinoma ovary
b) Carcinoma cervix
c) Carcinoma endometrium
Correct Answer - C
Ans. is 'c' i.e., Carcinoma endometrium
Staging of Carcinoma endometrium:
Though surgical staging is recommended, clinical staging is
applicable in operable cases.
A staging laparotomy is recommended through a midline lower
abdominal incision and the peritoneal ascetic fluid on washings is
collected for cytology.
Complete abdominal exploration followed by total abdominal
hysterectomy (TAH) along with bilateralsalpingo-oophorectomy
(BSO) omentectomy and pelvic and para-aortic lymph node
sampling remains the cornerstone in the management of early
endometrial cancer
149. T 1/2 of oxytocin is -
a) 1 - 2 minutes
b) 3 - 4 minutes
c) 15 - 20 minutes
d) 25 - 30 minutes
Correct Answer - B
Ans. is 'b' i.e., 3 - 4 minutes
Oxytocin has a half life of 3-4 minutes and duration of action of
approximately 20 minutes.
150. If patient has acute salpingitis and
peritonitis, then what is the stage of PID?
a) Stage 1
b) Stage 2
c) Stage 3
d) Stage 4
Correct Answer - B
Ans. is 'b' i.e., Stage 2
Pelvic inflammatory disease (PID) implies inflammation of upper
genital tract involving the fallopian tubes as well as the ovaries.
The spectrum ranges from mild to moderate and severe PID.
Depending upon the severity of tubal damage, Gainesville has
described five stages of PID :
Stage I - Acute salpingitis without peritonitis
Stage II - Acute salpingitis with peritonitis
Stage III - Acute salpingitis with superimposed tubal occlusion or
tubo - ovarian complex
Stage IV -Ruptured tubo - ovarian abscess
Stage V - Tubercular salpingitis.
151. Hyperemesis gravidarum is maximum at
what gestational age?
a) 6 weeks
b) 9 weeks
c) 28 weeks
d) 36 weeks
Correct Answer - B
Ans. is 'b' i.e., 9 weeks
Maximum levels of beta HCG occur on 66th day of pregnancy, i.e. 9
weeks 3 days, and thus hyperemesis is maximum at 9 weeks of
gestation.
152. Frequency of Braxton Hicks contraction
is -
a) One every 2 minutes
Correct Answer - C
Ans. is 'c' i.e., One every 15 minutes
Braxton - Hicks contractions
Uterine contractions in pregnancy has been named after Braxton -
Hicks who first described this entity.
These contractions are irregular, infrequent, spasmodic and painless
without any effect on dilatation of cervix.
Patient is not conscious about the contractions.
The intensity varies between 5 and 25 mmHg.
The number increases during the last week or two, when the uterus
may contract as often as every 10 to 20 minutes.
153. Correct order in labour -
i) Flexion ii) Crowning
iii) External rotation iv) Restitution
a) i - - - iv
b) i-ii-iv-iii
c) ii - iii - iv - i
d) ii-iv-i-iii
Correct Answer - B
Ans. is 'b' i.e., i - ii - iv - iii
Descent of head in normal labour follows these steps in the
following order :
Engagement
Increasing flexion
Internal rotation of occiput anteriorly to 2/8th of circle.
Simultaneous rotation of the shoulders to 1/8th of circle.
Crowning
Delivery of the head by extension
Restitution
External rotation
Delivery of the shoulders and trunk by lateral flexion.
154. Chance of subsequent baby having NTD
when first baby was born with NTD -
a) 2%
b) 4%
c) 10%
d) 20%
Correct Answer - B
Ans. is 'b' i.e., 4%
The recurrence risk of neural tube defect is :
3 - 5 % if a couple has previously had a child with either
anencephaly or spina bifida.
5% if either parent was born with a NTD.
10% if a couple has two affected children
155. Treatment of choice for sarcoma
botryoides is
a) Surgical excision
b) Radio therapy
c) Chemotherapy
d) Palliative therapy
Correct Answer - C
Ans is 'c' i.e., Chemotherapy
Sarcoma Botryoides
Sarcoma botryoides is a rare tumour seen in children.
This tumour arises in the mesenchymal tissues of the vagina and in
rare cases, in the cervix before the age of 2 years.
It presents as a haemorrhagic grape-like polyp or as a fleshy mass
and consists of rhabdomyoblasts with vacuolated cytoplasm,
myxoedema and stroma with fusiform cells.
The tumour spreads by local infiltration, lymphatics and blood
stream.
Examination is done under anaesthesia; biopsy confirms the
diagnosis. CT and MRI indicate its spread.
Treatment. Chemotherapy with VAC (vincristine, adriamycin and
cyclophosphamides) is the gold standard in treating this tumour.
Other drugs used are cisplatin, actinomycin, cyclophosphamide and
ifosfamide.
Surgery is limited to the local residual tumour. Intersti- tial radiation
is used in advanced stage.
156. Which of the following is true about
wandering fibroid?
a) Attached to uterus
Correct Answer - C
Ans. is 'c' i.e., Attached to surrounding viscera
A subserous pedunculated myoma may complicate by undergoing
rotation at the site of its attachment to the uterus.
As a result, its veins are occluded and the tumour becomes
engorged with blood.
Patient presents with very severe abdominal pain.
In very rare cases, these rotated tumors may adhere to an adjacent
viscera and obtain a fresh blood supply from these adhesions and
finally be detached completely from the uterus and are attached only
to adjacent viscera.
Such fibroids are called wandering or parasitic fibroids.
157. Nitabuch'slayer is absent is
a) Placenta accrete
b) Placenta previa
c) Placenta membranacia
d) Circumvallate placenta
Correct Answer - A
Ans is 'a' i.e., Placenta accrete
Nitabuchs layer
During formation of placenta, there is an area of fibrinoid
degeneration where trophoblast cells (covered with syncytium) meet
the decidua.
This zone is known as Nitabuch's layer. This layer limits further
invasion of the decidua by the trophoblast. Note: This membrane is
absent in placenta accreta.
158. Cervical pregnancy is confirmed by the
presence of
a) Gestational sac below internal os
c) Bleeding is painless
Correct Answer - D
Ans is 'd' i.e., Histology showing presence of villi inside the
cervical stroma
Cervical Pregnancy
In cervical pregnancy, the bleeding is painless and the uterine body
lies above the distended cervix. Intractable bleeding following
evacuation or expulsion of the products brings about suspicion.
The morbidity and mortality is high because of profuse hemorrhage.
Clinical diagnostic criteria (Rubin-1983) for cervical pregnancy are?
Soft, enlarged cervix equal to or larger than the fundus.
Uterine bleeding following amenorrhea, without cramping pain.
Products of conception entirely confined within and firmly attached to
endocervix.
A closed internal cervical os and a partially opened external os.
Sonography reveals the pregnancy in the cervical canal and an
empty uterine cavity.
Confirmation is done by histological evidence of the presence of villi
inside the cervical stroma.
159. Which of the following is included in the
III step of management of atonic uterus?
a) IV Calcium Gluconate
c) Balloon tamponade
Correct Answer - B
Ans is 'b' i.e., Uterine Massage and bimanual compression
STEPS :
I Massage uterus, injection oxytocin, foleys catheter insertion to
keep bladder empty, examine expelled placenta and membranes
II Exploring uterus under GA, IM inj 15 methyl PGE2 alpha, per
rectal PGE1, IV calcium gluconate if atony secondary to tocolytic
agent)
III Uterine Massage and bimanual compression
IV Uterine tamponade - tight uterine packing, balloon tamponade.
160. Air embolism occurs in which method of
abortion?
a) Spontaneous abortion
c) Criminal abortion
Correct Answer - C
Ans is 'c' i.e., Criminal abortion
In criminal abortion death may occur from shock, hemorrhage, air or
fat embolism and sepsis. When poisonous substances have been
administered death may supervene from their effects.
DIC and cerebral damage may follow when abortion is induced by
intrachorionic injection of hypertonic saline and glucose after 12th
week.
161. In Procidentiawhich of the following is
true?
a) Uterus and cervix in vagina
Correct Answer - C
Ans is 'c' i.e., Both uterus and vagina outside the introitus
Uterine descent
Descent of the cervix into the vagina.
Descent of the cervix up to the introitus.
Descent of the cervix outside the introitus.
Procidentia - All of the uterus outside the introitus
Procidentia:
Procidentia refers to the complete prolapse beyond the level of the
hymen distally so the uterus (or vaginal vault if uterus is absent) is
permanently protruding out of the vagina.
Women with procidentia have poor coordination of pelvic muscle
relaxation and contraction, associated bowel dysfunction, and
underlying urinary incontinence.
INVESTIGATIONS:
The investigations required with procidentia are few, but renal
ultrasonography and mid-stream urinalysis may be important, owing
to potential for kinking of the ureters leading with consequent
hydronephrosis and urine stasis due to incomplete emptying.
In addition, some authors have suggested urodynamic investigations
prior to surgery to help identify occult stress urinary incontinence
and to attempt to predict those women with underlying voiding
dysfunction.
MANAGEMENT:
Surgical options for procidentia include: vaginal hysterectomy with
anterior and posterior traditional colporrhaphy; or vaginal
hysteropexy utilising uterosacral and cardinal ligament complex;
sacrospinous fixation; or abdominal or laparoscopic hysteropexy,
with or without mesh.
162. All the following represent risk factors
associated with pelvic inflammatory
disease except
a) Recent new sexual partner
b) Doughing
d) Age 30 - 39 years
Correct Answer - D
Ans is 'd' i.e., Age 30 - 39 year
Risk factors associated with pelvic inflammatory disease are :
Douching
Single status
Substance abuse
Multiple sexual partners
Lower socio - economic status
Recent new sexual partner
Young age 10 - 19 years
Other sexually transmitted infections
Sexual partner with urethritis or gonorrhea
Previous diagnosis of pelvic inflammatory disease
Not using mechanical or chemical contraceptive barriers
Endocervical testing positive for N. gonorrhea or C. trachomatis
163. Nodular tags of hymen in the post
pregnancy period are called
a) Carunculaemyrtiformes
b) Vestibulaemyrtiformes
c) Orficiaemyrtiformes
d) Carunculaeorificies
Correct Answer - A
Ans is 'a' i.e., Carunculaemyrtiformes
The hymen can stretch or tear as a result of various behaviours,
by tampon or menstrual cup use, pelvic examinations with
a speculum, regular physical activity, sexual intercourse, insertion of
multiple fingers or items into the vagina, and activities such as
gymnastics (doing 'the splits'), or horseback riding.Remnants of the
hymen are called carunculaemyrtiformes.
164. Volume of ovary after menopause is -
a) 7.8 cm3
b) 6.5 cm3
c) 5.4 cm3
d) 3.0 cm3
Correct Answer - D
Ans. is 'd' i.e.,3.0 cm3
Volume of ovary can be calculated based on a simple formula:
Volume = 1/2 x Length x height x width
Ovarian Volume in women
Mean ovarian
Age
vol.
0 0.2
2 0.7
4 0.9
6 1.2
8 1.7
10 2.5
12 3.7
14 5.0
16 6.4
18 7.3
20 7.7
22 7.6
24 7.2
26 6.7
28 6.3
28 6.3
30 6.0
32 5.9
34 5.9
36 5.8
38 5.7
40 5.4
42 4.9
44 4.4
46 3.8
48 3.3
50 2.8
165. Average reproductive life span of ovum
is?
a) 6-12 hrs
b) 12-24 hrs
c) 24-36 hrs
d) 3 days
Correct Answer - B
Ans. is 'b' i.e.,12-24 hrs
Reproductive life of ovum → 24 hours (12 - 24 hours)
Reproductive life of sperm → 24 - 72 hours
166. Arterial embolization done in all except ?
a) Uterine AV malformation
b) PPH
c) Bleeding fibroid
Correct Answer - D
Ans. is 'd' i.e., Prevent bleeding in endometriosis
Indications for embolization in Obstetrics and Gynecology are
the following:
Postpartum hemorrhage: Uterine atony / tears, Abnormal
placentation
Post abortion/ ectopic
Post hysterectomy
Uterine AV malformation/ Gestational trophoblastic tumors
Gynecological malignancies
Fibroids (acute bleeding)
167. Small size placenta is seen in ?
a) Peripheral vascular disease in mother
b) Pre eclampsia
d) Multiple pregnancy
Correct Answer - B
Ans. is 'B' i.e., Pre eclampsia[Ref Internet]
Small placenta on antenatal ultrasound USG may be due to :-
Variation in placental morphology :Only post of placenta is seen.
Bibbed placenta :Only one lobe is seen.
Succenturiate lobe.
Hypertensive states (pre-eclampsia, eclampsia)
Chromosomal disorders :Trisomy 18, Dysgenic triploidy, IUGR or
intrauterine infection
168. Following features are suggestive of
severe eclampsia except?
a) Blood pressure > 150/100 mmhg
d) Thrombocytopenia
Correct Answer - A
Ans. is 'a' i.e., Blood pressure > 150/100
Preeclampsia is considered severe if one or more of the
following criteria is present:
Blood pressure of 160 mm Hg systolic or higher or 110 mm Hg
diastolic or higher on two occasions at least 6 hours apart while the
patient is on bed rest.
Proteinuria of 5 g or higher in a 24-hour urine specimen or 3+ or
greater on two random urine samples collected at least 4 hours
apart.
Oliguria of less than 500 ml in 24 hours
Cerebral or visual disturbances
Pulmonary edema or cyanosis
Epigastric or right upper-quadrant pain
Impaired liver function
Thrombocytopenia
Fetal growth restriction
169. Vulvar carcinoma accounts for what
percentage of genital tract
malignancies?
a) 0.5 - 1%
b) 3 - 5%
c) 7 - 11%
d) 13 - 15%
Correct Answer - B
Ans. is 'b' i.e.,3 - 5%
Vulval Carcinoma
It occurs in 17/100,000 females.
It accounts for 3 - 5% of the genital tract malignancies
It usually occurs in post menopausal females with median age of 60
years.
It has increased association with obesity, hypertension, diabetes and
multiparity.
170. Most common histopathological variety
of carcinoma vulva ?
a) Squamous cell carcinoma
c) Adenocarcinoma
d) Lymphoma
Correct Answer - A
Ans. is 'a' i.e.,Squamous cell carcinoma
Squamous cell carcinomas are the most common histological type
accounting for 85% of all vulval cancers.
171. Maximum teratogenic effect on heart of
fetus is between?
a) 2 - 4 weeks
b) 4 - 6 weeks
c) 6 - 8 weeks
d) 8 - 10 weeks
Correct Answer - C
Ans. is 'c' i.e.,6 - 8 weeks
Period of organogenesis (Embryonic - 2 - 8 weeks of conception) is
the time of maximum vulnerability, and each embryonic system has
a period of maximum susceptibility.
Heart is typically affected if the teratogen acts between 6 - 8 weeks
of gestation.
CNS maximum susceptibility between 17 - 30th post fertilization
days.
172. Pregnant women with obesity are at
higher risk of following except?
a) Fetal macrosomia
b) Fetal distress
c) Gestational hypertension
d) Infections
Correct Answer - D
Ans. is 'd' i.e.,Infections
In a prospective multicenter study, pregnant females with
obesity (BMI of 30-39.9) was associated with an increased risk
of the following compared to non obese females:
Gestational diabetes mellitus
Preeclampsia
Gestational hypertension
Fetal macrosomia
173. Which method of contraception is not
used in patient with active syphilis?
a) IUCD
c) Barrier devices
d) Calendar method
Correct Answer - A
Ans. is 'a' i.e.,IUCD
Syphilis is a sexually transmitted disease.
Use of IUCD is strictly contraindicated in patient with active genital
tract infection thus use of IUCD is contraindicated in in patient with
active syphilis
174. Gardasil protects against following HPV
strains except ?
a) 6
b) 16
c) 18
d) 33
Correct Answer - D
Ans. is 'd' i.e.,33
HPV Vaccines
Cervarix - Protects against HPV-16, 18.
Gardasil 4 Protects against HPV - 6, 11, 16, 18.
Gardasil -9 - Protects against HPV - 6, 11, 16, 18, 31, 33, 45, 52, 58.
175.
Common indications for caesarian section in
primigravidae are all except ?
a) Failed induction
b) Cephalopelvic disproportion
c) Dystocia
d) Malpresentation
Correct Answer - D
Ans. is 'd' i.e., Malpresentation
Primigravidae
Failed induction
Fetal distress (non-reassuring FHR)
Cephalo-pelvic disproportion
Dystocia
Malpresenation, malposition
Multigravidae
Previous caesarean delivery
APH
Malpresenation
176. Fibroid with maximum symptoms is ?
a) Submucous
b) Subserous
c) Intramural
d) Cervical
Correct Answer - A
Ans. is 'a' i.e., Submucous
177. Twin pregnancy of the same age and sex
rules out ?
a) Superfetation
b) Maternal twins
c) Superfecundation
Correct Answer - A
Ans. is 'a' i.e., Superfetation
Superfetation
It is the simultaneous occurrence of more than one stage of
developing offspring in the same animal.
In mammals, it manifests as the formation of an embryo from a
different estrous cycle while another embryo or fetus is already
present in the uterus.
178. At what level of testosterone ovarian
pathology should be searched ?
a) > 4 nmol/L
b) > 6 nmol/L
c) > 8 nmol/L
d) >10 nmol/L
Correct Answer - D
Ans. is 'd' i.e.,> 10 nmol/L
Testosterone in females
It is secreted by the ovary (60%) and also derived from the
peripheral conversion of androstenidione (40%), which is secreted in
equal amounts by the ovary and adrenals.
179. Increased blood pressure for > 3 months
after delivery ?
a) Pregnancy induced hypertension
b) Essential hypertension
c) Pre eclampsia
d) Gestational hypertension
Correct Answer - B
Ans. is 'b' i.e., Essential hypertension
Diagnostic criteria for essential hypertension in pregnancy
Rise in BP >140/ 90 mm Hgprior to 20th week of pregnancy.
Cardiac enlargement on chest radiograph and ECG.
Presence ofmedical disorders.
Prospective follow up shows rise in BP ever after 42 days following
delivery.
180. Which of the following is false about
gestational hypertension?
a) There is a sustained rise of BP > 140/90 mmHg
Correct Answer - C
Ans. is 'C i.e., It is associated with lower incidence of essential
hypertension in the later life as compared to pre eclampsia
Gestational hypertension
A sustained rise of blood pressure to 140/90 mm Hg or more on
atleast 2 occasions 4 or more hours apart beyond the 20thweeks of
pregnancyor during the first 24 hours after delivery in a previously
normotensive woman is called gestational hypertension.
It is associated much higher incidence of essential hypertension in
later life than pre eclampsia.
The hypertensive effect is actually astressresponse.
Perinatal mortality remains unaffected in a case of gestational
hypertension.
The patients with gestational hypertension are more likely to develop
hypertensionwith the use of oral contraceptives or in sub sequent
pregnancies.
B.P. returns to normal within 6 weeks of delivery
181. Surgery in genital TB Is not indicated in -
a) Pyometra
c) Pyosalpinx
Correct Answer - B
Ans. is'b'i.e., Plastic bowel adhesions
Indications for surgery in case of genital tuberculosis:
Progression of disease
Persistent activelesion despite chemotherapy
Persistence of large inflammatory masses i. e. pyosalpinx, Pyometra
despite chemotherapy
Persistence of symptoms i. e. pain, menorrhagia despite
chemotherapy
Persistence offistula despite chemotherapy
Contraindications for surgery in case of genital tuberculosis:
Active lesions elsewhere in body
Plastic adhesions ofbowel
182. 29 years female, G2 P1 A1 with 16 weeks
amenorrhoea, lower abdominal pain and
vaginal bleeding, ultrasonography shows
snow storm appearance. What is the
most feasible method of treatment
a) Misoprostol administration
b) Hysterectomy
c) Mifepristone
d) Suction evacuation
Correct Answer - D
Ans. D. Suction evacuation
29 yrs female with amenorrhea of 16 week with lower abdominal
pain and vaginal bleeding and snow storm appearance on USG
gives us a diagnosis of hydatidiform mole.
The most preferred definitive treatment in case of hydatidiform mole
in almost all cases is suction evacuation.
It can be easily done upto 28 week of gestation.
In favourable cervix suction evacuation is done with a negative
pressure of 200 - 250 mm Hg.
The cervix is closed prior dilatation isfollowed by suction and
evacuation.
183. Zygote is dependent on which of the
following for its nutrition -
a) Deutoplasm
Correct Answer - D
Ans. is 'd' i.e., All the above
While zygote is passing down the fallopian tube and after a brief
period as it enters the uterus, it depends for its nutrition on the yolk
sac granules (deutoplasm) embedded in its cytoplasm and on the
fluid medium surrounding it which is secreted by the walls of the
uterine tube and uterus.
It also derives its energy from the stored carbohydrate in sperm,
whichfertilized the ovum,
184. For salpingitis true is
a) Always bilateral
Correct Answer - D
Ans. is 'd' i.e., Bacterial vaginosis is not a risk factor for pelvic
inflammatory disease development
Pelvic inflammatory disease (PlD)
It is the infection of upper female reproductive tract organs.
As fallopian tube are involved most commonly, it is also called as
acute salpingitis.
It is associated most commonly with N gonorrhoea and chlamydial
infection.
Bacterial vaginosis is not a risk factor for PID.
PID / acute salpingitis may be unilateral or bilateral.
SequelaeofPlDare:-
1. Tubal factor infertility
2. Ectopic pregnancy
3. Chronic Pelvic Pain
185. Fibromyoma is seen in age group
a) Reproductive
b) Early adolescence
c) Latepostmenopausal
d) Prepubertal
Correct Answer - A
Ans. is'a'i.e., Reproductive
Fibroids are the most common benign solid tumours in females.
It is the most common Pelvic tumour
Most commonly affects the reproductive age group.
Most commonly seen in nulliparous common
186. Most common infection associated with
IUCD use is
a) Chlamydia
b) Staphylococcus epidermidis
c) Staphylococcus aureus
d) Group D Streptococcus
Correct Answer - A
Ans. a.Chlamydia
Infections with chlamydia and actinomyces are most common with
use of IUCD
187. Condition not responsive to medical
therapy
a) Invasive mole
b) Choriocarcinoma
c) Hydatidiform mole
Correct Answer - C
Ans. is'c' i.e., Hydatidiform mole
It is an abnormal condition o[ the placenta where there are partly
degenerative and partly proliferative changes in the young chorionic
villi.
It is best regarded benign neoplasia of the chorion with rnalignant
potential.
Management: Suction evacuation
188. Mucosal lining of fallopian tube secretion
for ovum nutrition controlled by -
a) LH
b) FSH
c) Estrogen
d) Progesterone
Correct Answer - D
Ans. D. Progesterone
The secretions of the fallopian tube are a mode of nutrition to the
developing zygote till, it reaches uterus and placenta forms.
Progesterone increases the secretion of the fallopian tube mucous
membrane and. abo causes hyperplasia of the muscular lining of the
fallopian tube and makes its peristaltic movements stronger
189. Fourchette is formed by?
a) Joining of labia majora
Correct Answer - B
Ans. is'b'i.e., Joining of labia minora
Fourchette :
The fourchette is a thin fold of skin, identified when the labia are
separated, and it is often torn during parturition.
The fossa navicularis is the small hollow between the hymen and the
fourchette.
The labia minora lie between the labia majora, with which they
merge posteriorly, and are separated into two folds as they
approach the clitoris anteriorly.
The anterior folds unite to form the prepuce or hood of the clitoris.
The posterior folds form the frenulum ofthe clitoris.
Inferiorly, the labia minora extend to approach the midline as low
ridges of tissue that fuse to from the fourchette
190. Malaria in pregnancy doesn't cause?
a) HELLP
b) IUGR
c) IUD
d) Preterm
Correct Answer - A
Ans. is'a'i.e., HELLP
Effect of malaria on pregnancy:
IUGR
Abortion
IUD
Preterm
Low birthweight
191. Gland homologous to prostate in
females is?
a) Gartner's gland
b) Skene's gland
c) Bartholin's gland
d) Cowper's gland
Correct Answer - B
Ans. B. Skene’s gland
In female human anatomy, Skene's glands or the Skene glands
known as the lesser vestibular glands, paraurethral glands, or
homologous femaleprostate are glands located on the anterior wall
of thevagina, around the lower end of the urethra.
192. Incomplete uterine rupture is defined as?
a) Disruption ofpart ofscar
Correct Answer - D
Ans. is'd' i.e., Disruption of scar with peritoneum intact
Incomplete rupture :
In an Incomplete uterine rupture, the mother’s peritoneum remains
intact.
The peritoneum is the membrane that lines the abdominal cavity to
support abdominal organs.
It also acts as a channel for blood vessels and nerves.
An incomplete uterine rupture is significantly less dangerous with
fewer complications to the delivery process.
Complete rupture:
During a Complete uterine rupture, the peritoneum tears and the
contents of the mother’s uterus can spill into her peritoneal cavity.
The peritoneal cavity is the fluid-filled gap that separates the
abdomen walls and its organs.
It is suggested that delivery via cesarean section (C- section) should
occur within approximately 10 to 35 minutes after a complete uterine
rupture occurs.
The fetal morbidity rate increases dramatically after this period.
193. Cells seen at the function between two
layers ofplacenta are?
a) Hofbauer cell
b) Hofmann cells
c) Amniogenic cells
Correct Answer - A
Ans. is'a'i.e., Hofbauer cell
Hofbauer cells (HBCs) are placental macrophages that are present
in the core of villus.
Major cell type in placenta include syncytiotrophoblasts which line
intervillous space and are in direct contact of maternal blood.
Underlying stromal cells adjacent to fetal capillaries largely
consisting off bro blasts and Hofbauer cells (fetal tissue
macrophages)
194. Which of the following is not a pre-
requisits for transvaginal sonography
(TVS)?
a) Consent
b) Fullbladder
c) Empty bladder
d) Lithotomyposition
Correct Answer - B
Ans. B. Full bladder
Transvaginalsonographyisbest performedwith an emptybladder,
which enables the pelvic organs to reach acloser proximity to the tip
of the high-frequency transvaginal probe.
On the other hand trans abdominal sonography is done with full
bladder to get an overview of anatomy.
195. Which of the following is not true of
placenta?
a) Number of cotyledons increases with gestational age
Correct Answer - D
Ans. is'd'i.e., At term about four fifths of placenta of maternal
origin
Placenta
Human placenta is discoid in shape.
It has two surfaces, fetal and maternal.
In the beginning the placenta growth is more rapid than that of the
fetus, by 17th week they have equal weight.
By term placental weight is one sixth of fetal weight.
Maternal surface constitute 15-20 somewhat polygonal placental
lobule, or cotyledon.
Each lobule is supplied with a single truncal branch of chorionic
artery.
And each lobule has a single vein so that so that lobules constitute
functional units of placental architecture.
Placenta at term
At term it is almost circular disc with a diameter of 15-20 cm and
thickness about 3 cm at center.
lt's approximate weight is 500 gm.
The ratio of placental: fetal weight is 1: 6 and occupies about 30% of
uterine wall.
At term, about four fifths of placenta is fetal in origin.
196. Most common histological variety of
Uterine carcinoma is?
a) Squmous cell carcinoma
c) Adeno carcinoma
d) Mixed carcinoma
Correct Answer - C
Ans. is'c'i.e., Adenocarcinoma
Carcinoma of uterus :
Endometrial carcinoma is the most common invasive cancer of the
female genital tract and accounts for 7% of all invasive cancer in
women.
On histologic examination, most endometrial carcinomas (about
85%) are adenocarcinomas characterized by more or less well-
defined gland patterns closely resembling normal endometrial
epithelium.
197. Male factor is responsible for infertility in
how much percentage -
a) 5%
b) 20%
c) 30%
d) 50%
Correct Answer - C
Ans. is'c'i.e., 30%
Infertility:
Male factor: 30%
Tubal, uterine & peritoneal factor: 25%
Ovarian factor: 25%
Cervical factor: 10%
Unexplained factor: 10%
198. In hypergonadotropic hypogonadism
FSH level is?
a) <20 m IU/ ml
b) <40 m IU/ ml
c) >20 m IU/ ml
d) >40 m IU/ ml
Correct Answer - D
Ans. is'd'i.e., > 40 m IU/ml
FSH LEVEL:
Age 0-7 years: <6.7 m IU/ml
Age >7 years:
Follicular phase: 3.1 -7.9 ml IU/ml
Ovulation peak : 2.3 – 18.5 ml IU/ml
Luteal phase :1.4- 5.5 ml IU/ml
Postmenopausal: 30.6-106.3 ml IU/ml
Hypergonadotropic hypogonadism: >40ml IU/ml
199. Nerve commonly injured in forceps
delivery is ?
a) Common peroneal
b) Obturator
d) Sciatic
Correct Answer - B
Ans. is'b'i.e., Obturator
Obturator nerve injury ( L2-4) may result during forceps delivery in
lithotomy position resulting from acute Flexion of hip,
hematoma,trauma from forceps blade.
200. Dysmenorrhoea is due to ?
a) Ovulation
b) Decreased progesterone
c) Increasedprogesterone
d) Secretoryepithelium
Correct Answer - B
Ans. is'b'i.e., Decreased progesterone
Dysmenorrhoea refers to painful cramping pain accompanying
menstruation.
Dysmenorrhoea in ovulatory cycle is due to release of PGs, in
endometrium induced by progesterone
Dysmenorrhoea is classified into :
Primary dysmenorrhoea refers to one that is not associated with any
identifiable pelvic pathology. It is now clear that the pathogenesis of
pain is attributable to a biochemical derangement.
Secondary dysmenorrhoea refers to the one associated with the
presence of organic pelvic pathology, i.e. fibroids, adenomyosis, PID
endometriosis. Unilateral dysmenorrhoea occurs in a rudimentary
horn of a bicornuate uterus